PDA

View Full Version : [Official Thread]Richieste d'aiuto in MATEMATICA: postate qui!


Pagine : 1 2 3 4 5 6 7 8 9 10 11 12 13 14 15 16 17 18 19 20 21 22 23 24 [25] 26 27 28 29 30 31 32

Jarni
13-12-2009, 22:34
Up

Altro ex:
Esprimere il numero z0 = ( 1+i )/(1-i) nella forma canonica e risolvi (z0)^(1/4) { radice alla quarta di z0 ).
La prima parte credo di averla fatta giusta:
razionalizzo moltiplicando per (1+i) sia sopra che sotto, ottengo:
[(1+i)^2]/2
Svolgo il quadrato ottenendo: (1+2i-1)/2
Semplifico quello che c'è da moltiplicare ed ottengo che z0 = i

La seconda parte dell'esercizio richiede, in buona sostanza, di trovare la forma esponenziale di z0. Da qui non sono sicuro di procedere in modo corretto...
Pongo z0^(1/4), quindi i^(1/4)
So che z0 = rho*e^(i*teta)
Dove rho = sqrt(x^2+y^2), quindi rho = 1
Teta = arctg(y/x) = arctg(1) = 45° = pi.gr/4
z0 = e^(i * pi.gr/4 )
Corretto?

A parte che sbagli l'angolo:
x=Re[z0]=0
y=Im[z0]=1
Teta = arctg(Im[z0]/Re[z0]) = arctg(1/0) = arctg(infinito) = 90° = pi.gr/2
z0 = e^(i * pi.gr/2)

Ora viene il difficile.
Sai che ogni angolo alfa è equivalente ad alfa+2*pi.gr, o ad alfa+2*2*pi.gr, ad alfa+3*2*pi.gr...
Calcolare la radice quarta di e^(i * pi.gr/2) è come calcolare la radice quarta di z0 = e^(i * (pi.gr/2+2*pi.gr)), ecc... MA NON E' DETTO CHE I RISULTATI SIANO GLI STESSI. Quindi inseriamo questo fattore additivo nell'angolo di fase.
Quindi:

z0 = e^(i * (pi.gr/2+n*2*pi.gr)) dove n è un numero intero

allora

z0^(1/4)=e^(i * (pi.gr/2+n*2*pi.gr)/4)

Vediamo che angolo esce in quell'esponente:
(pi.gr/2+n*2*pi.gr)/4=(pi.gr+4*n*pi.gr)/8=pi.gr*(1+4*n)/8

Quindi il risultato sarebbe e^(i*(1+4*n)/8): dipende da n.
Ma questo non significa che la radice quarta di z0 abbia infiniti valori.
Dopo un certo n, gli angoli calcolati ricalcheranno i precedenti.
Se infatti li elenchiamo:

n=0--->l'angolo è pi.gr/8
n=1--->l'angolo è (5*pi.gr)/8
n=2--->l'angolo è (9*pi.gr)/8
n=3--->l'angolo è (13*pi.gr)/8
n=4--->l'angolo è (17*pi.gr)/8

Ma attenzione: l'ultimo angolo non è altro che

(17*pi.gr)/8=(16*pi.gr)/8+(pi.gr)/8=2*pi.gr+(pi.gr)/8

che ci riporta a pi.gr/8, quindi da n=4 in poi non faremo altro che ripercorrere i seguenti angoli ciclicamente:

pi.gr/8
(5*pi.gr)/8
(9*pi.gr)/8
(13*pi.gr)/8

Ed infatti, la radice quarta di z0 ha QUATTRO valori:

e^(i*(pi.gr/8))
e^(i*(5*pi.gr)/8)
e^(i*(9*pi.gr)/8)
e^(i*(13*pi.gr)/8)

kwb
14-12-2009, 10:07
Ciao, ho provato a guardare quello che dici tu e quello che avevo scritto sugli appunti.
Non capisco una cosa, noi sappiamo che la radice quarta di n è uguale a n^(1/4), quindi so che rho^(1/4) = 1, quindi rho = 1
Questo risultato lo devo mettere a sistema con n*teta = pi.gr/8 + 2k*pi.gr quindi
Sistema tra:
rho^(1/4) =1 rho = 1
teta/4 = pi.gr/8 + 2k*pi.gr teta = (pi.gr/2) + 8k*pi.gr

Non capisco come faccia a venire quello che hai scritto tu....

Jarni
14-12-2009, 13:15
Ciao, ho provato a guardare quello che dici tu e quello che avevo scritto sugli appunti.
Non capisco una cosa, noi sappiamo che la radice quarta di n è uguale a n^(1/4), quindi so che rho^(1/4) = 1, quindi rho = 1

No, no. Riguarda bene la teoria sul libro, perché è un discorso un po' sottile.;)
Rho è il modulo di z0, non z0.
La seconda parte dell'esercizio ti chiede di trovare la radice quarta di z0, del numero complesso, non del modulo rho(ebbene sì, si può fare la radice di un numero complesso:D ).

Per fare questo serve esprimere z0 in notazione esponenziale.

Ogni numero complesso è esprimibile come rho*e^(i*theta), dove rho è un numero reale positivo o nullo(la distanza tra l'origine degli assi e il punto z0 nel piano di Gauss) e theta(reale pure lui) è l'angolo compreso tra l'asse orizzontale del p. di Gauss e la retta che congiunge z0 con l'origine. Ovviamente theta va da 0 a 2*pi.

Proprio perché stiamo operando con numeri complessi, l'aspetto più inconsueto è che la radice n-esima di un numero complesso da' sempre n risultati diversi(in fondo anche la radice quadrata di 9 da' più di un risultato: 3 e -3...).

Il motivo di ciò è dovuto alla presenza di una quantità angolare. E' difficile da spiegare a parole, ma segui questo esempio, che dovrebbe rendere l'idea.

Supponi di avere un'orologio, con la lancetta dei minuti sulle 12.
Ora, vuoi spostare la lancetta di un certo angolo per quattro volte di seguito, usando sempre lo stesso angolo, in modo da ritornare esattamente al punto di partenza.
Potresti usare un angolo di 90°, infatti 90+90+90+90=360, che ti riporta alla posizione iniziale.
Però potresti usare anche un angolo di 180°. Infatti 180+180+180+180=720, che sono due angoli giri, e quindi ritorni alla posizione originaria.
Lo stesso se usi un angolo di 270°: 270*4=1080°, sono tre angoli giri, ritorni sempre alla posizione iniziale.
Un'altro angolo banale è 0°: "non fai nulla" per 4 volte, ritorni a dove eri partito.
Come vedi, hai trovato 4 angoli diversi(0, 90, 180 e 270) che se applicati 4 volte ti riportano alla posizione iniziale. E' immediato verificare, poi, che non ce ne sono altri, minori di 360°, che ti permettono questo risultato.
Matematicamente, questo è come dire che la radice quarta di un numero complesso ha 4 risultati diversi.:read:

Se vuoi la radice terza troverai gli angoli 0, 120 e 240...
Se vuoi la radice quinta: 0, 72, 144, 216, 288...:sofico:

kwb
14-12-2009, 14:11
Si, questo discorso è chiaro.
Ci siamo solo capiti male, io quando dico rho = 1 intendo rho = sqrt( x^2 + y^2), siccome nel mio caso x = 0 e y = 1 ottengo che rho = sqrt(0^2 + 1^2) = 1 .
Visto che rho è uguale a 1, che sia sotto radice quinta, decima o ventundicesima ( :sofico: ) sempre 1 sarà.
Io di solito opero in questo modo:
Trovo prima rho
Trovo l'angolo theta ( facendo l'arcotangente di y/x )
Metto a sistema rho e theta con le seguenti condizioni:
Rho^n = quello che ho trovato
n*theta = 2k*pi.gr + l'angolo theta trovato

Così facendo non dovrei riuscire a trovare la giusta notazione esponenziale del numero complesso?

Inoltre, per capire quanti sono i K, non mi basta guardare a cosa è elevata la funzione ( in questo caso 4, quindi K = 0, 1, 2, 3 )?

Altra cosa: se mi viene chiesto di definire il massimo intervallo nel quale una data funzione è invertibile, mi basta fare la derivata prima e porla maggiore di 0, guardare dove cresce e dove decresce e stabilirne quindi l'intervallo max?

Jarni
14-12-2009, 18:40
Si, questo discorso è chiaro.
Ci siamo solo capiti male, io quando dico rho = 1 intendo rho = sqrt( x^2 + y^2), siccome nel mio caso x = 0 e y = 1 ottengo che rho = sqrt(0^2 + 1^2) = 1 .
Visto che rho è uguale a 1, che sia sotto radice quinta, decima o ventundicesima ( :sofico: ) sempre 1 sarà.
Io di solito opero in questo modo:
Trovo prima rho
Trovo l'angolo theta ( facendo l'arcotangente di y/x )
Metto a sistema rho e theta con le seguenti condizioni:
Rho^n = quello che ho trovato
n*theta = 2k*pi.gr + l'angolo theta trovato

Così facendo non dovrei riuscire a trovare la giusta notazione esponenziale del numero complesso?

Se fai i conti trovi i miei stessi risultati, avrai fatto un errore di calcolo.:confused:

Riprendiamo bene le definizioni.
Hai un numero z0, che in notazione di Eulero è
z0=rho*e^(i*theta)=1*e^(i*pi/2)
Devi trovare un numero z1 che è la radice quarta di z0.
z1=z0^(1/4)
E la notazione esponenziale di z1 è rho1*e^(i*theta1).
Adesso, sai che
z1^4=z0
questo comporta che
rho1^4=rho=1 e quindi rho1=1
ma anche che
4*theta1=theta+2k*pi=pi/2+2k*pi

Con pochi calcoli trovi
theta1=pi/8+k*pi/2

k=0->theta1=pi/8
k=1->theta1=pi/8+pi/2=5*pi/8
k=2->theta1=pi/8+2*pi/2=9*pi/8
k=3->theta1=pi/8+3*pi/2=13*pi/8
Quindi hai quattro valori per z1:

z1=e^(i*pi/8)
z1=e^(i*5*pi/8)
z1=e^(i*9*pi/8)
z1=e^(i*13*pi/8)


Inoltre, per capire quanti sono i K, non mi basta guardare a cosa è elevata la funzione ( in questo caso 4, quindi K = 0, 1, 2, 3 )?
Certo.;)


Altra cosa: se mi viene chiesto di definire il massimo intervallo nel quale una data funzione è invertibile, mi basta fare la derivata prima e porla maggiore di 0, guardare dove cresce e dove decresce e stabilirne quindi l'intervallo max?
Una funzione è invertibile in tutti i punti dove la derivata è diversa da 0.
Gli intervalli saranno compresi tra due punti che annullano la derivata prima, oppure tra un punto e infinito, oppure tra -infinito e +infinito...
Non capisco il senso di "massimo" intervallo.:mbe:

kwb
14-12-2009, 19:05
Se fai i conti trovi i miei stessi risultati, avrai fatto un errore di calcolo.:confused:

Riprendiamo bene le definizioni.
Hai un numero z0, che in notazione di Eulero è
z0=rho*e^(i*theta)=1*e^(i*pi/2)
Devi trovare un numero z1 che è la radice quarta di z0.
z1=z0^(1/4)
E la notazione esponenziale di z1 è rho1*e^(i*theta1).
Adesso, sai che
z1^4=z0
questo comporta che
rho1^4=rho=1 e quindi rho1=1
ma anche che
4*theta1=theta+2k*pi=pi/2+2k*pi

Con pochi calcoli trovi
theta1=pi/8+k*pi/2

k=0->theta1=pi/8
k=1->theta1=pi/8+pi/2=5*pi/8
k=2->theta1=pi/8+2*pi/2=9*pi/8
k=3->theta1=pi/8+3*pi/2=13*pi/8
Quindi hai quattro valori per z1:

z1=e^(i*pi/8)
z1=e^(i*5*pi/8)
z1=e^(i*9*pi/8)
z1=e^(i*13*pi/8)


Certo.;)
Ok, grazie, ora è chiaro.



Una funzione è invertibile in tutti i punti dove la derivata è diversa da 0.
Gli intervalli saranno compresi tra due punti che annullano la derivata prima, oppure tra un punto e infinito, oppure tra -infinito e +infinito...
Non capisco il senso di "massimo" intervallo.:mbe:
Appunto quindi fai la derivata maggiore di 0 e con lo schemino trovi dove cresce e dove decresce.
Per massimo intervallo credo si intenda una roba del genere: poniamo che la funzione esiste nell'intervallo [0;+inf)
Cresce nell'intervallo [0;5)
Decresce nell'intervallo (5;7)

Se andiamo a vedere l'ampiezza di di piano comperta dai 2 intervalli scopriamo che il primo è maggiore del secondo, infatti 5-0 = 5 e 7-5 = 2
Penso si debba fare una cosa del genere, cmq quello ceh mi interessa è sapere se fare la derivata prima > 0 è corretto. Tutto qua. Per questo posso anche chiedere al professore all'esame.

misterx
14-12-2009, 20:03
ciao,
il mio precedente post non vi è piaciuto, forse troppo banale la domanda ? :stordita:

Provo con questa:


lim log(1 + x^2) - sen(x)^2
x->0 -----------------------
(1 - cos(x) + x^4)^2


vorrei risolverlo senza usare Tatyor oppure De l'Hopital ma solo attraverso limiti notevoli e gli infinitesimi

Il risultato lo conosco già grazie a Derive, mi interessano i passaggi per la semplificazione.

grazie

diablo...aka...boss
14-12-2009, 20:31
Qualcuno mi dice come si risolvono gli integrali perfavore :cry: io non me lo ricordo più e da google wikipedia & co. non c'ho capito na cippa :muro:
grazie in anticipo a chiunque possa essermi d'aiuto.

misterx
14-12-2009, 21:00
Qualcuno mi dice come si risolvono gli integrali perfavore :cry: io non me lo ricordo più e da google wikipedia & co. non c'ho capito na cippa :muro:
grazie in anticipo a chiunque possa essermi d'aiuto.

guarda a questo link http://www.ripmat.it/mate/c/ck/ck.html

clasprea
14-12-2009, 21:39
sto provando a fare un esercizio sugli integrali doppi ma mi blocco ad un certo punto, e a dirla tutta non sono nemmeno sicura che sia giusto il pezzo che ho già fatto :lamer:

per di più di solito in questi esercizi in cui ci sono due integrali uno è nullo, ma non so dire qual è dei due :doh:

vi prego, aiutate una gentil donzella in difficoltà! :flower:

http://img96.imageshack.us/img96/7154/img017m.th.jpg (http://img96.imageshack.us/i/img017m.jpg/)

Jarni
15-12-2009, 04:02
Se andiamo a vedere l'ampiezza di di piano comperta dai 2 intervalli scopriamo che il primo è maggiore del secondo, infatti 5-0 = 5 e 7-5 = 2
Penso si debba fare una cosa del genere, cmq quello ceh mi interessa è sapere se fare la derivata prima > 0 è corretto. Tutto qua. Per questo posso anche chiedere al professore all'esame.

Per l'invertibilità non ti serve sapere se la funzione cresce o decresce, basta che la derivata non sia nulla.
Io porrei la derivata =0, risolvo l'equazione e trovo tutti i punti di non invertibilità...

regshout
15-12-2009, 10:15
Qualcuno può spiegarmi, anche via pvt, equazioni di primo grado, disequazioni, e sistemi??
Perchè non ci sto capendo niente...

misterx
15-12-2009, 10:36
Qualcuno può spiegarmi, anche via pvt, equazioni di primo grado, disequazioni, e sistemi??
Perchè non ci sto capendo niente...

vai a questo link http://www.ripmat.it/mate/a/af/af.html

Ziosilvio
15-12-2009, 14:16
sto provando a fare un esercizio sugli integrali doppi ma mi blocco ad un certo punto, e a dirla tutta non sono nemmeno sicura che sia giusto il pezzo che ho già fatto :lamer:

per di più di solito in questi esercizi in cui ci sono due integrali uno è nullo, ma non so dire qual è dei due :doh:

vi prego, aiutate una gentil donzella in difficoltà! :flower:

http://img96.imageshack.us/img96/7154/img017m.th.jpg (http://img96.imageshack.us/i/img017m.jpg/)
Il primo è l'integrale, su un dominio simmetrico rispetto all'asse delle ordinate, di una funzione che è dispari nella variabile x.
Per cui...

Il secondo integrando è simmetrico sia rispetto alla x che rispetto alla y, quindi puoi calcolare solo sul "ramo destro" e poi raddoppiare.

clasprea
15-12-2009, 14:26
Il primo è l'integrale, su un dominio simmetrico rispetto all'asse delle ordinate, di una funzione che è dispari nella variabile x.
Per cui...

Il secondo integrando è simmetrico sia rispetto alla x che rispetto alla y, quindi puoi calcolare solo sul "ramo destro" e poi raddoppiare.

ok, grazie mille, per cui il primo si può dire che è nullo. L'impostazione che ho fatto per il primo è giusta? Posso replicarla per risolvere il secondo?

Ziosilvio
15-12-2009, 17:52
ok, grazie mille, per cui il primo si può dire che è nullo. L'impostazione che ho fatto per il primo è giusta? Posso replicarla per risolvere il secondo?
Mi sembra che l'idea di fondo sia quella giusta.

clasprea
15-12-2009, 18:36
Mi sembra che l'idea di fondo sia quella giusta.

grazie

PacManZ
15-12-2009, 21:27
Curiosità..

date f(x) e g(x) soluzioni di un'equazione differenziale del secondo ordine, è possibile scrivere l'equazione differenziale che ha f e g soluzioni?

Jarni
16-12-2009, 11:23
Curiosità..

date f(x) e g(x) soluzioni di un'equazione differenziale del secondo ordine, è possibile scrivere l'equazione differenziale che ha f e g soluzioni?

Certo: scrivi l'equazione iniziale.:D
A parte gli scherzi, non mi è chiaro cosa intendi: fai un esempio.

diablo...aka...boss
16-12-2009, 11:37
guarda a questo link http://www.ripmat.it/mate/c/ck/ck.html

grazie.

PacManZ
16-12-2009, 16:29
Certo: scrivi l'equazione iniziale.:D
A parte gli scherzi, non mi è chiaro cosa intendi: fai un esempio.


se voglio scrivere un'equazione di secondo grado che ha a e b come soluzioni mi basta scrivere (x-a)(x-b)=0, cioe' x^2-(a+b)x+ab=0..

Esiste un metodo per scrivere un'equazione differenziale che ha f(x) e g(x) come soluzioni?

Jarni
16-12-2009, 18:04
se voglio scrivere un'equazione di secondo grado che ha a e b come soluzioni mi basta scrivere (x-a)(x-b)=0, cioe' x^2-(a+b)x+ab=0..

Esiste un metodo per scrivere un'equazione differenziale che ha f(x) e g(x) come soluzioni?

(y'-f'(x))*(y'-g'(x))=0 :D

PacManZ
16-12-2009, 18:48
ma y'*y' è uguale a y''?

Xalexalex
16-12-2009, 19:47
ma y'*y' è uguale a y''?

Una velocità al quadrato è uguale ad un'accelerazione? :D

PacManZ
16-12-2009, 21:22
Una velocità al quadrato è uguale ad un'accelerazione? :D



quindi la formula che mi hanno dato sopra non vale

Jarni
16-12-2009, 23:27
quindi la formula che mi hanno dato sopra non vale

Non sarà del secondo ordine, ma vale eccome.

Ileana
17-12-2009, 09:42
Ho da affrontare un problema di teoria delle code.

E' stato osservato un fenomeno e ne sono stati rilevati i tempi fra due arrivi. Il dominio di appartenenza di tali arrivi è stato suddiviso in classi con ampiezza pari a 20 minuti.

http://img109.imageshack.us/img109/7660/tcode.th.jpg (http://img109.imageshack.us/i/tcode.jpg/)

QUi ci sono i dati della tabella (il + indica il valore inferiore escluso).
C'è la formula che ho nella parte di teoria, poi c'è la formula che è presentata nell'esercizio.


Ora, sono gnucca. Perchè è diversa (errore o va bene lo stesso?), inoltre, come faccio a farmi venire il risultato di 2,0346 clienti/h? ._.

Vi prego è abbastanza urgente ._.

misterx
17-12-2009, 12:51
ciao,
dovendo lavorare con i limiti ci si scontra sempre con infiniti/infinitesimi e mi chiedevo se questa classifica vi pare corretta considerando la direzione verso +infinito:

dal più veloce al più lento

1) x^x
2) x!
3) e^x
4) x^n
5) x
6) sqrt(x)
7) log(x)

grazie

kwb
17-12-2009, 15:34
ciao,
dovendo lavorare con i limiti ci si scontra sempre con infiniti/infinitesimi e mi chiedevo se questa classifica vi pare corretta considerando la direzione verso +infinito:

dal più veloce al più lento

1) x^x
2) x!
3) e^x
4) x^n
5) x
6) sqrt(x)
7) log(x)

grazie
Ti direi di si, l'unico di cui non so il comportamento e la rapidità di aumento è x^x .
Da x! in giù sono corretti

Ziosilvio
17-12-2009, 16:08
ciao,
dovendo lavorare con i limiti ci si scontra sempre con infiniti/infinitesimi e mi chiedevo se questa classifica vi pare corretta considerando la direzione verso +infinito:

dal più veloce al più lento

1) x^x
2) x!
3) e^x
4) x^n
5) x
6) sqrt(x)
7) log(x)

grazie
x! non è definito per x non intero; però lo è http://operaez.net/mimetex/%5CGamma(x+1), dove http://operaez.net/mimetex/%5CGamma è la funzione gamma di Eulero; ed è noto che http://operaez.net/mimetex/%5CGamma(n+1)=n! se n è un numero naturale,
Ti direi di si, l'unico di cui non so il comportamento e la rapidità di aumento è x^x .
Da x! in giù sono corretti
L'ordine di infinito è corretto. Puoi valutare facilmente che n!/n^n < 1/n per ogni n>2.

misterx
17-12-2009, 18:55
ciao,
grazie per la risposta, e chiaramente verso zero l'ordine si inverte ?

ordine degli infinitesimi: dal più lento a più veloce verso lo zero

7) x^x
6) x!
5) e^x
4) x^n
3) x
2) sqrt(x)
1) log(x)


http://it.wikipedia.org/wiki/Stima_asintotica#Confronti_fra_infiniti_e_infinitesimi

sekkia
17-12-2009, 19:20
Secondo me l'ordine rimane sempre quello.
Ad esempio, prendendo x e sqrt(x) abbiamo:
x = 0,01
sqrt(x) = 0,1
Quindi x tende a 0 più velocemente della sua radice quadrata.
Allo stesso modo:
e^0,001 = 1,0010005
0,001^0,001 = 0,993116048
Quindi x^x tende a 0 più velocemente di e^x.
Non sono sicuro, ma penso che l'ordine rimanga invariato rispetto agli infiniti. :stordita:

misterx
17-12-2009, 19:37
Secondo me l'ordine rimane sempre quello.
Ad esempio, prendendo x e sqrt(x) abbiamo:
x = 0,01
sqrt(x) = 0,1
Quindi x tende a 0 più velocemente della sua radice quadrata.
Allo stesso modo:
e^0,001 = 1,0010005
0,001^0,001 = 0,993116048
Quindi x^x tende a 0 più velocemente di e^x.
Non sono sicuro, ma penso che l'ordine rimanga invariato rispetto agli infiniti. :stordita:

magari sbaglio, ma prova con x=0,0000000000000001 e sqrt(0,0000000000000001)

lim(x/sqrtx,x,0) = 0
x->0

se provi tutti quegli infinitesimi tenendo al denominatore logx tendono tutti a zero.

Ziosilvio
18-12-2009, 06:04
ciao,
grazie per la risposta, e chiaramente verso zero l'ordine si inverte ?

ordine degli infinitesimi: dal più lento a più veloce verso lo zero

7) x^x
6) x!
5) e^x
4) x^n
3) x
2) sqrt(x)
1) log(x)


http://it.wikipedia.org/wiki/Stima_asintotica#Confronti_fra_infiniti_e_infinitesimi
Non è così semplice, perché l'esponenziale, la gamma di Eulero traslata, e l'"autoesponenziale" convergono a 1 per x che tende a 0.

ServiceXone
18-12-2009, 14:23
piccola domanda

cosa significa l'uguale concavo convesso ? :stordita:

http://www.pctunerup.com/up/results/_200912/20091218151836_pict0001.jpg
(per x --> 0)

asintotico a zero ?

Ziosilvio
18-12-2009, 15:37
piccola domanda

cosa significa l'uguale concavo convesso ? :stordita:

http://www.pctunerup.com/up/results/_200912/20091218151836_pict0001.jpg
(per x --> 0)

asintotico a zero ?
Dovrebbe voler dire che il rapporto delle due grandezze si mantiene limitato in un intorno dell'origine.

jacky guru
18-12-2009, 16:27
Le mie modeste conoscenze mi suggeriscono che quel simbolo indichi "equigrande" o "dello stesso ordine", riferito a due funzioni il cui rapporto, al limite (dunque per x che tende a qualcosa, in un I insomma...) è un numero reale non necessariamente pari a 1 ma diverso da 0 (nel caso sia 1 si parla di funzioni "equivalenti" e si indica con una tilde).

kierlo
18-12-2009, 18:16
Quel simbolo significa che hanno eguale ordine di grandezza, ossia:
Esistono due costanti c,d>0 t.c per un intorno di raggio delta:

0<d <= | f(x) / g(x)| <= c

Ossia che il rapporto fra le due funzioni per x che tende a p (d'accumulazione) è compreso fra due costanti.

Ovviamente x=!p (diverso)

Scusate se non ho usato il latex ma vado di fretta e non ho la mano..

jacky guru
18-12-2009, 20:19
Un dubbio che mi perseguita ormai da due anni: perchè l'integrale di 1/x è il logaritmo naturale del VALORE ASSOLUTO di x?! Insomma, questo valore assoluto a cosa serve?

Ad esempio svolgendo questo integrale per parti:

ln(x+3) / x^2

mi ritrovo come primitiva:

-1/x[ln(x+3)] + [ln |x|]/ 3 - [ln|x+3|]/3

Insomma se volessi compattare i logaritmi... come trattare i valori assoluti?

kwb
18-12-2009, 20:51
Penso che sia dovuto al fatto che il logaritmo esiste solo per valori di x positivi ( maggiori di 0 ).

kierlo
19-12-2009, 12:41
puoi usare la proprietà dei logaritmi del quoziente ed avere solo un valore assoluto.

jacky guru
19-12-2009, 12:49
puoi usare la proprietà dei logaritmi del quoziente ed avere solo un valore assoluto.

Ci ho pensato, tuttavia ho alcuni termini con valore assoluto ed altri senza... :confused:

Perocchio
19-12-2009, 18:13
Ho bisogno del vostro aiuto per risolvere questi 2 problemi di probabilità, spero possiate darmi una mano quanto prima:
1)Un vagone è formato da n scompartimenti che ospitano n passeggeri.Se salgono n passeggeri che scelgono a caso il posto, con quale probabilità nessuno scompartimento resterà vuoto?
2) Un condensatore ceramico ha una capacità(espressa in microfarad) che è una variabile aleatoria Y=3600\X con X una variabile Gaussiana con media=10000 e scarto tipo =500. Se vengono estratti in maniera s-indipendente 10 condensatori qual'è la probabilità che almeno 3 dei condensatori estratti abbiano capacità minore di 0.34286?

Datami una mano!!!

Perocchio
19-12-2009, 18:58
nessuno??

Ziosilvio
19-12-2009, 19:40
1. Non più di un "up" al giorno.
2. Unisco al thread delle richieste di aiuto in Matematica.

misterx
20-12-2009, 12:05
ciao,
ho iniziato a studiare gli integrali e stavo osservando il seguente esercizio:

--
| sen^2(x) dx
| ---
| 2
--


tale integrale viene trasformato e posto uguale al seguente:


--
| 1 - cos x dx
| ---------
| 2
--


poi vengono applicate le regole di integrazione.
Quello che mi chiedo è se la trasformazione passa per la bisezione e siccome nella formula della bisezione si vede una radice, mi chiedo se sia stata semplificata al volo e cioè, il mio libro ha saltato un passaggio.

grazie

balint
20-12-2009, 12:08
La "trasformazione" consiste proprio nell'applicazione della formula di bisezione, cioè:

http://operaez.net/mimetex/sinx%20=%20sqrt%28%281-cos2x%29/2%29

Nel passaggio che hai riportato la radice è stata già semplificata col quadrato, tuttavia c'è un errore, al secondo passaggio è http://operaez.net/mimetex/cos%282x%29.

misterx
20-12-2009, 12:12
La "trasformazione" consiste proprio nell'applicazione della formula di bisezione, tuttavia c'è un errore, al secondo passaggio è cos(2x).

scusa, mancava il denominatore :stordita:
Ma la radice che fine ha fatto nella trasformazione, è stata semplificata al volo ?

mi rispondo da solo e credo proprio di si anche se on conosce quelle formule

balint
20-12-2009, 13:16
La radice è andata via con il quadrato.

misterx
20-12-2009, 15:38
La radice è andata via con il quadrato.

grazie per la conferma

ciao

misterx
21-12-2009, 12:16
ciao,
scusate ma chiedo lumi su una derivata!

y=4x^2 * cos(4x^2 + 6x +2)

y'=8x * cos(4x^2 + 6x +2) - 4x^2 * sen(4x^2 +6x +2) * 8x + 6

cosa ho dimenticato ?

Ho usato il prodotto di derivate e la derivata di funzione di funzione

y'=f'(x)*g(x) + f(x) * h'(g(x)) * g'(x)

BlackJack84
21-12-2009, 12:20
y'=8x * cos(4x^2 + 6x +2) - 4x^2 * sen(4x^2 +6x +2) * (8x + 6)

cosa ho dimenticato ?


La parentesi! :D

misterx
21-12-2009, 12:34
La parentesi! :D

ma è giusta ? :stordita:

ilguercio
21-12-2009, 17:54
ma è giusta ? :stordita:

Si,pare di si.
Mi passo la palla con una domanda:
Ho un ombrello bianco che devo schermare dal lato convesso con un panno nero(magari poi spiego i motivi).
Faccio un esempio:
http://www.amiciebici.it/rob/immagini/gadget/ombrello-1.jpg
Solo che il mio è un pò più "tondo" diciamo.
Domanda,come faccio a conoscere la proiezione su un piano del tessuto che devo ritagliare per coprire l'ombrello?
Non so se mi spiego:se doveste coprire l'esterno dell'ombrello con del tessuto tagliato da un solo lenzuolone di stoffa che disegnereste sul lenzuolo da tagliare?
Insomma,vi chiedo lo sviluppo su un piano della superficie esterna dell'ombrello.
Non mi interessa essere troppo preciso,però meglio andare per eccesso che per difetto(per ovvi motivi).

ilguercio
23-12-2009, 18:05
Nada?:stordita:

kierlo
23-12-2009, 18:21
Partirei dal fatto che ha piu' assi di simmetria e farei la superficie di ognugno per 8. Ma ti serve un telo unico o puoi dividerlo a fette e poi unirle? così sarebbe piu' comodo..altrimenti nn saprei di preciso.

T3d
23-12-2009, 18:29
mi serve la lunghezza dell'arco creato dalla punta dell'ombrello fino ad una estremità, la lunghezza tra la punta dell'ombrello e il piano definito dalla circonferenza del ombrello e la lunghezza tra il palo dell'ombrello e una delle estremità dell'ombrello.

ilguercio
23-12-2009, 19:18
mi serve la lunghezza dell'arco creato dalla punta dell'ombrello fino ad una estremità, la lunghezza tra la punta dell'ombrello e il piano definito dalla circonferenza del ombrello e la lunghezza tra il palo dell'ombrello e una delle estremità dell'ombrello.

Tempo di prendere il metro da sarta,grazie;)

ilguercio
23-12-2009, 20:42
La lunghezza dell'arco creato dalla punta dell'ombrello fino ad una estremità:60cm
La lunghezza tra la punta dell'ombrello e il piano definito dalla circonferenza del ombrello:29-30cm
La lunghezza tra il palo dell'ombrello e una delle estremità dell'ombrello:50cm

Fai conto che sono misure prese col metro da sarta,meglio una stima in eccesso che una in difetto:)
Non deve essere precisissimo,mi va bene uno schemino con le misure "abbondanti".
Grazie:)

T3d
23-12-2009, 21:06
Fai conto che sono misure prese col metro da sarta,meglio una stima in eccesso che una in difetto:)
Non deve essere precisissimo,mi va bene uno schemino con le misure "abbondanti".
Grazie:)

allora, per avere un cono (non esattamente una calotta sferica) con la stessa area della superficie esterna dell'ombrello, devi prendere una circonferenza di stoffa di raggio 60cm e tagliare una porzione di circonferenza con un arco di 21 cm. poi la chiudi e trovi un cono che approssima l'ombrello.

si può fare di meglio, magari ragliando non proprio una porzione intera di circonferenza, ma fermandosi prima.

ilguercio
23-12-2009, 21:50
Mmm,credo di avere capito,se ci spreco la stoffa te la tiro dietro:D
L'importante è che copra,ripeto,non devo farci un vestito aderente.
Se mi dici che funziona domani taglio:)

T3d
23-12-2009, 23:34
Mmm,credo di avere capito,se ci spreco la stoffa te la tiro dietro:D
L'importante è che copra,ripeto,non devo farci un vestito aderente.
Se mi dici che funziona domani taglio:)

ehi, lo sai che la teoria si verifica sperimentalmente :D

comunque se vuoi tagliare in difetto, aumenta di qualche cm (magari 3cm) il raggio della circonferenza e diminuisci di qualche cm (altri 3cm) l'arco da tagliare.

ilguercio
23-12-2009, 23:39
ehi, lo sai che la teoria si verifica sperimentalmente :D

comunque se vuoi tagliare in eccesso, aumenta di qualche cm (magari 3cm) il raggio della circonferenza e diminuisci di qualche cm (altri 3cm) l'arco da tagliare.

Grazie ancora,domani provo:)

jacky guru
26-12-2009, 08:10
Ciao ragazzi!
Innanzitutto auguri a tutti! :)

Domanda: se ho due matrici quadrate tali che A*B=B*A, tale condizione implica sempre e soltanto che A (e B) siano invertibili e che una è l'inversa dell'altra? Faccio notare che ho scritto A*B=B*A, e non A*B=B*A=I...

kierlo
26-12-2009, 12:23
Se ricordo bene B è l'inversa di A proprio perchè è univocamente determinata.
Inoltre se moltiplichi a sinistra per A inversa e a destra per A hai la matrice identica.

Però non sono sicuro perchè non trovo gli appunti -.-

T3d
26-12-2009, 14:30
Ciao ragazzi!
Innanzitutto auguri a tutti! :)

Domanda: se ho due matrici quadrate tali che A*B=B*A, tale condizione implica sempre e soltanto che A (e B) siano invertibili e che una è l'inversa dell'altra? Faccio notare che ho scritto A*B=B*A, e non A*B=B*A=I...

se AB=BA, allora A è simile a B

Ziosilvio
26-12-2009, 15:37
se ho due matrici quadrate tali che A*B=B*A, tale condizione implica sempre e soltanto che A (e B) siano invertibili e che una è l'inversa dell'altra? Faccio notare che ho scritto A*B=B*A, e non A*B=B*A=I...
No, in generale due matrici possono commutare senza che una sia l'inversa dell'altra.
Pensa solo a questo: ogni multiplo della matrice identità commuta con ogni matrice.

Xalexalex
26-12-2009, 15:48
Curiosità che mi è venuta studiando: conoscete per caso una dimostrazione del teorema di Weierstrass che non richieda la conoscenza del teorema di Bolzano e nemmeno la nozione di "compattezza" di un inseme? Ovviamente non vale far finta di non conoscere il teorema di Bolzano e poi ridimostrarlo mentre si dimostra Weierstrass :sofico:

Ziosilvio
26-12-2009, 19:55
Curiosità che mi è venuta studiando: conoscete per caso una dimostrazione del teorema di Weierstrass che non richieda la conoscenza del teorema di Bolzano e nemmeno la nozione di "compattezza" di un inseme? Ovviamente non vale far finta di non conoscere il teorema di Bolzano e poi ridimostrarlo mentre si dimostra Weierstrass :sofico:
Temo di no: se ci pensi, il teorema di Weierstrass è il caso particolare, relativo alla retta reale, del principio per cui l'immagine continua di un compatto è compatta.

jacky guru
27-12-2009, 11:25
se AB=BA, allora A è simile a B
mmh sicuro? che io sappia A è simile a B se esiste una matrice P, invertibile, tale che A = Pinversa*B*P
No, in generale due matrici possono commutare senza che una sia l'inversa dell'altra.
Pensa solo a questo: ogni multiplo della matrice identità commuta con ogni matrice.
Ok grazie ;)

Seconda domandina: se in un punto la derivata prima vale zero, anche la sua derivata seconda, se f(x) è C2 nel suo dominio, varrà zero, vero?

Ziosilvio
27-12-2009, 12:42
se in un punto la derivata prima vale zero, anche la sua derivata seconda, se f(x) è C2 nel suo dominio, varrà zero, vero?
Naturalmente no: pensa al caso in cui f(x) è un polinomio di secondo grado con due radici distinte. Allora f è C-infinito sulla retta reale, la derivata prima si annulla esattamente in un punto, e la derivata seconda è costante e non nulla.

jacky guru
27-12-2009, 12:54
Mmh capisco... la mia domanda veniva dallo svolgimento di una dimostrazione applicando il teorema di Rolle.

In particolare, mi si dice che esiste una funzione di classe C(2) nell'intervallo I = (-5, 0) e f(-4)=f(-2)=f(-1)=6 ; devo dimostrare che esiste almeno un punto appartenente a tale intervallo la cui derivata SECONDA vale zero.

Dunque ho iniziato considerando il sottointervallo [-4, -2] in cui f(-4)=f(-2). In tale intervallo la funzione è continua così come la sua derivata prima e seconda. E' anche derivable, dunque applicando il teorema di Rolle trovo che esiste un punto la cui derivata prima vale 0.

Tuttavia mi si chiede di determinare un punto la cui derivata SECONDA è zero! :confused:

misterx
28-12-2009, 19:10
ciao,
siccome ho un docente che quando svolge gli esercizi da tutto per scontato mi vedo costretto a chiedere lumi :stordita:

Data la disequazione:

|3+x| <= 1 / |1+x|

ho individuato gli intervalli sulla retta reale dove fare i conti e cioè:
|3+x|= 3+x se x >= -3
|3+x|= -3-x se x < -3

|1+x|= 1+x se x >= -1
|1+x|= -1-x se x < -1

i tre intervalli rappresentati sulla retta reale sono:

-oo ---------------- -3 ------------- -1 -------------- +oo

quindi:
1) x < -3
2) -3 <= x < -1
3) x < -1

per x < -3

-3-x <= 1 / -1-x

porto tutto al primo membro e faccio il m.c.m


(-3-x)(-1-x)
------------ <= 0
-1-x

x^2 + 4x +4
------------ <= 0
-1-x


determini le radici del numeratore ed ottengo
x1=-2+\/2
x2=-2-\/2

Domanda
Siccome il docente fa sempre sparire il denominatore moltiplicando 1° e 2° membro per il denominatore stesso, dicendo che intanto sappiamo di stare operando nel campo negativo mi chiedo: volendo calcolare anche il denominatore e sufficiente scrivere:
-1-x=0 quindi x=-1

quindi rappresentare sul grafico le radici del numeratore e denominatore tenendo in considerazione solo l'ntervallo x < -3 ?

---------- (-2-\/2 ) ------- -3 --------------- -1 ----------------

In questo caso, siccome -1 non cade nell'intervallo che sto studiando x < -3 lo sto calcolando inutilmente ?

il risultato del primo caso sarebbe -2-\/2 <= x < -3

grazie

Jarni
29-12-2009, 02:17
ciao,
siccome ho un docente che quando svolge gli esercizi da tutto per scontato mi vedo costretto a chiedere lumi :stordita:

Data la disequazione:

|3+x| <= 1 / |1+x|

ho individuato gli intervalli sulla retta reale dove fare i conti e cioè:
|3+x|= 3+x se x >= -3
|3+x|= -3-x se x < -3

|1+x|= 1+x se x >= -1
|1+x|= -1-x se x < -1

i tre intervalli rappresentati sulla retta reale sono:

-oo ---------------- -3 ------------- -1 -------------- +oo

quindi:
1) x < -3
2) -3 <= x < -1
3) x < -1

per x < -3

-3-x <= 1 / -1-x

porto tutto al primo membro e faccio il m.c.m


(-3-x)(-1-x)
------------ <= 0
-1-x

x^2 + 4x +4
------------ <= 0
-1-x


determini le radici del numeratore ed ottengo
x1=-2+\/2
x2=-2-\/2

Domanda
Siccome il docente fa sempre sparire il denominatore moltiplicando 1° e 2° membro per il denominatore stesso, dicendo che intanto sappiamo di stare operando nel campo negativo mi chiedo: volendo calcolare anche il denominatore e sufficiente scrivere:
-1-x=0 quindi x=-1

quindi rappresentare sul grafico le radici del numeratore e denominatore tenendo in considerazione solo l'ntervallo x < -3 ?

---------- (-2-\/2 ) ------- -3 --------------- -1 ----------------

In questo caso, siccome -1 non cade nell'intervallo che sto studiando x < -3 lo sto calcolando inutilmente ?

il risultato del primo caso sarebbe -2-\/2 <= x < -3

grazie

1) x < -3
2) -3 <= x < -1
3) x < -1

Se sei nel caso 1), il denominatore -1-x è sempre una quantità positiva.
Consideralo come tale.;)

Il fatto è che stai studiando il segno di una frazione definita in una regione di R tale che il denominatore è sempre positivo, quindi è inutile calcolarne il segno... lo conosci già.

Se invece dovessi risolvere in tutto R questa frazione:

x^2 + 4x +4
------------ <= 0
-1-x

allora dovresti studiare il segno di numeratore e denominatore.

P.S.
Hai detto questo:

-3-x <= 1 / -1-x

porto tutto al primo membro e faccio il m.c.m
Il risultato non è questo:

(-3-x)(-1-x)
------------ <= 0
-1-x

Ma questo

(-3-x)(-1-x)-1
--------------- <= 0
-1-x

misterx
29-12-2009, 08:11
ciao,
e grazie per la risposta :)
Volendo calcolare il denominatore ugualmente anche se non sto considerando tutto R ma da x < -3 quando faccio il grafico dovrei ottenere lo stesso risultato giusto ?

Marcko
29-12-2009, 10:06
Salve, ho un problema con le equazioni lineari del secondo ordine non omogenee a coefficienti costanti. Dunque, fintato si tratta di un termine noto del tipo http://operaez.net/mimetex/f%28x%29=ax%5E2+bx+c, il problema non si pone, ma nel momento in cui si tratta di un termine noto del tipo http://operaez.net/mimetex/f%28x%29=e%5E%7B%5Clambda%20x%7D, non si capisce più nulla sul mio libro.
Conoscete dispense, libro o altro che possa aiutarmi, vi ringrazio.
Marco

T3d
29-12-2009, 11:22
http://ocw.mit.edu/NR/rdonlyres/E1B388FB-CE4F-4732-B7A4-04DC1D9FF5D5/0/ode_review.pdf

questo è tutto quello che ti serve :D

Marcko
29-12-2009, 11:24
http://ocw.mit.edu/NR/rdonlyres/E1B388FB-CE4F-4732-B7A4-04DC1D9FF5D5/0/ode_review.pdf

questo è tutto quello che ti serve :D

Accidenti all'inglese :muro: :D
Grazie mille ora stampo e leggo.

misterx
29-12-2009, 17:49
ciao,
necessitavo più chealtro di una conferma.
Data la disequazione

|3+x| <= 1 / |1+x|

e individuati gli intervalli da studiarsi sulla retta reale e cioè:

|3+x| = 3+x se x >= -3
|3+x| = -3-x se x < -3

|1+x| = 1+x se x >= -1
|1+x| = -1-x se x < -1

-oo -------------- (-3) ------------------ (-1) ------------- +oo

quindi 3 intervalli:
1) x < -3
2) -3 <= x < -1
3) x > -1

se io faccio lo studio del segno normalmente valutando numeratore e denominatore ogni volta nei 3 casi e di volta in volta escludo le soluzioni esterne all'intervallo che sto considerando dovrei ottenere i risultati corretti giusto ?

Nel primo caso ad esempio ho x < -3 e facendo i conti ottengo:

x1= -2+\/2
x2= -2-\/2

siccome sto lavorando per x <-3 il risultato x1 non lo considero giusto ?
In tale caso ho che il risultato del primo caso è: -2-\/2 <= x < -3

grazie e auguri

Jarni
29-12-2009, 18:58
ciao,
necessitavo più chealtro di una conferma.
Data la disequazione

|3+x| <= 1 / |1+x|

e individuati gli intervalli da studiarsi sulla retta reale e cioè:

|3+x| = 3+x se x >= -3
|3+x| = -3-x se x < -3

|1+x| = 1+x se x >= -1
|1+x| = -1-x se x < -1

-oo -------------- (-3) ------------------ (-1) ------------- +oo

quindi 3 intervalli:
1) x < -3
2) -3 <= x < -1
3) x > -1

se io faccio lo studio del segno normalmente valutando numeratore e denominatore ogni volta nei 3 casi e di volta in volta escludo le soluzioni esterne all'intervallo che sto considerando dovrei ottenere i risultati corretti giusto ?

Nel primo caso ad esempio ho x < -3 e facendo i conti ottengo:

x1= -2+\/2
x2= -2-\/2

siccome sto lavorando per x <-3 il risultato x1 non lo considero giusto ?
In tale caso ho che il risultato del primo caso è: -2-\/2 <= x < -3

grazie e auguri

x1 sta fuori dell'intervallo di definizione, quindi non non ne devi tener conto.

misterx
29-12-2009, 19:06
x1 sta fuori dell'intervallo di definizione, quindi non non ne devi tener conto.

perfetto, grazie 1000

jacky guru
30-12-2009, 09:48
Mmh capisco... la mia domanda veniva dallo svolgimento di una dimostrazione applicando il teorema di Rolle.

In particolare, mi si dice che esiste una funzione di classe C(2) nell'intervallo I = (-5, 0) e f(-4)=f(-2)=f(-1)=6 ; devo dimostrare che esiste almeno un punto appartenente a tale intervallo la cui derivata SECONDA vale zero.

Dunque ho iniziato considerando il sottointervallo [-4, -2] in cui f(-4)=f(-2). In tale intervallo la funzione è continua così come la sua derivata prima e seconda. E' anche derivable, dunque applicando il teorema di Rolle trovo che esiste un punto la cui derivata prima vale 0.

Tuttavia mi si chiede di determinare un punto la cui derivata SECONDA è zero! :confused:
UP!! :help:

Seconda domanda: sto impazzendo nel trovare la primitiva di questo integrale ( :muro: ):

integrale di: 1 / (1 + x^120)

kwb
30-12-2009, 11:15
UP!! :help:

Seconda domanda: sto impazzendo nel trovare la primitiva di questo integrale ( :muro: ):

integrale di: 1 / (1 + x^120)
La primitiva che si avvicina di più è l'arcotangente ( la cui derivata è appunto 1/1+x^2 ). Forse per sostituzione si riesce a fare qualcosa, non so però....

jacky guru
30-12-2009, 18:42
La primitiva che si avvicina di più è l'arcotangente ( la cui derivata è appunto 1/1+x^2 ). Forse per sostituzione si riesce a fare qualcosa, non so però....
Ho provato, ma così facendo mi trovo, al numeratore, potenze indesiderate :(

Mixmar
30-12-2009, 19:46
Forse pensavi ad un altro integrale:

http://integrals.wolfram.com/index.jsp?expr=1+/+(1+%2B+x+)+^+120&random=false

piuttosto che:
http://integrals.wolfram.com/index.jsp?expr=1+/+(1+%2B+x+^+120)&random=false (http://integrals.wolfram.com/index.jsp?expr=1+/+(1+%2B+x+)+^+120&random=false)

:eekk: ?

kwb
30-12-2009, 20:29
Che differenza c'è tra il primo e il secondo link? :D

jacky guru
30-12-2009, 20:42
Forse pensavi ad un altro integrale:

http://integrals.wolfram.com/index.jsp?expr=1+/+(1+%2B+x+)+^+120&random=false

piuttosto che:
http://integrals.wolfram.com/index.jsp?expr=1+/+(1+%2B+x+^+120)&random=false (http://integrals.wolfram.com/index.jsp?expr=1+/+(1+%2B+x+)+^+120&random=false)

:eekk: ?
E secondo te in un tema d'esame di Analisi I del Politecnico di Torino mettono un simile integrale? Quel genere lì viene schifato pure dai libri di testo di analisi elle superiori :D

niente parentesi al denominatore ;)

misterx
31-12-2009, 09:53
edit

jacky guru
31-12-2009, 10:15
Mmh forse forse forse (ma mooolto forse) ci sono: in pratica mi si diceva di verificare SE la funzione integrale (tra 0 e x) di quell'integrale che ho postato prima ha un asintoto orizzontale a +oo.

Dunque ho pensato: se l'integrale converge la funzione integrale F(x) avrà un asintoto orizzontale... ed effettivamente applicando i criteri di convergenza, quella funzione è equivalente a 1/t^120 che a +oo (integrale improprio) CONVERGE.

Cosa ho sbagliato in questo ragionamento? :confused:

Mixmar
31-12-2009, 13:23
Che differenza c'è tra il primo e il secondo link? :D

E secondo te in un tema d'esame di Analisi I del Politecnico di Torino mettono un simile integrale? Quel genere lì viene schifato pure dai libri di testo di analisi elle superiori :D

niente parentesi al denominatore ;)

Ovviamente il secondo link era sbagliato: mi riferivo a questo:

http://integrals.wolfram.com/index.jsp?expr=1+/+(1+%2B+x+^+120)&random=false

Era per affermare che un integrale del genere difficilmente verrebbe proposto in un tema d'esame... comunque adesso vedo che non viene chiesto di calcolarlo esplicitamente, in realtà. Quindi probabilmente l'esercizio implicava rispondere al quesito senza risolvere l'integrale.

misterx
31-12-2009, 14:02
scusate ho sbagliato 3D

kwb
31-12-2009, 15:19
Si ho provato poi a sparare quell'integrale dentro quello script php, è spuntata fuori la soluzione e ho chiuso dalla paura :asd:

Domanda, ma si può fare uno sviluppo di taylor su una funzione dentro un integrale?

jacky guru
31-12-2009, 16:11
Che io sappia no, o meglio puoi usare Taylor per capire una data funzione, in un integrale improprio, a quale funzione potenza è equivalente, in modo da ricorrere al criterio del confronto asintotico per la determinazione del carattere di integrali impropri sia del I che del II tipo.

litocat
31-12-2009, 16:53
Domanda, ma si può fare uno sviluppo di taylor su una funzione dentro un integrale?
In generale no. Ricorda che una funzione f(x) non coincide necessariamente su tutto il suo dominio con la sua serie di Taylor: prendi atan(x), ha R come dominio ma la sua serie di Taylor ha come raggio di convergenza 1. Quindi se vuoi trovare \int_0^5 atan(x) dx non puoi certamente scrivere \int_0^5 atan(x) dx = \int_0^5 (\sum_{n=0}^\infty (-1)^n \frac{x^{2n+1}}{2n+1}) dx.

Se la funzione e' analitica e' tutta un'altra storia...

Ziosilvio
31-12-2009, 17:19
ma si può fare uno sviluppo di taylor su una funzione dentro un integrale?
In generale no. Ricorda che una funzione f(x) non coincide necessariamente su tutto il suo dominio con la sua serie di Taylor: prendi atan(x), ha R come dominio ma la sua serie di Taylor ha come raggio di convergenza 1. Quindi se vuoi trovare \int_0^5 atan(x) dx non puoi certamente scrivere \int_0^5 atan(x) dx = \int_0^5 (\sum_{n=0}^\infty (-1)^n \frac{x^{2n+1}}{2n+1}) dx.

Se la funzione e' analitica e' tutta un'altra storia...
Infatti il problema sta nell'uniforme convergenza della serie.
Se la serie converge uniformemente nel dominio di integrazione, allora la somma della serie degli integrali è pari all'integrale della somma della serie. Se la serie non converge uniformemente, questo potrebbe non succedere.

A tale proposito vale la pena di ricordare il criterio di convergenza delle serie di potenze:
Sia http://operaez.net/mimetex/%5Csum_{n=0}^{%5Cinfty}a_n(z-z_0)^n una serie di potenze e sia http://operaez.net/mimetex/R=1/%5Clim_{n%5Cto%5Cinfty}%5Csqrt[n]{|a_n|} il suo raggio di convergenza.
Se http://operaez.net/mimetex/R%5Cgt{0} allora la serie converge uniformemente in ogni compatto contenuto nel disco aperto di centro http://operaez.net/mimetex/z_0 e raggio http://operaez.net/mimetex/R.
Se http://operaez.net/mimetex/R%5Clt%5Cinfty allora la serie non converge in alcun punto http://operaez.net/mimetex/z tale che http://operaez.net/mimetex/|z-z_0|%5Cgt{R}

Fabietto206
03-01-2010, 15:21
Qualcuno riesce a spiegarmi i passaggi x risolvere la seguente sommatoria:

sommatoria per x=0 a n di: (e^t * p)^x * (1-p)^(n-x) * (n x)

ke deve venire: (e^t * p + 1 - p)^n


Ho anke il seguente limite:

lim di n -> +inf di: (1 - y/n)^n, perchè viene e^-y ??

Ziosilvio
03-01-2010, 15:31
Qualcuno riesce a spiegarmi i passaggi x risolvere la seguente sommatoria:
No: i passaggi per risolvere la sommatoria.
somm per x=0 a n di: (e^t * p)^x * (1-p)^(n-x) * (n x)

ke deve venire: (e^t * p + 1 - p)^n
No: che deve venire ecc.
Se quell'"(n x)" che leggo è un coefficiente binomiale, allora quella è un'applicazione della formula del binomio di Newton, che dovresti conoscere.
Ho anke il seguente limite:
No: anche il limite.
lim di n -> +inf di: (1 - y/n)^n, perchè viene e^-y ??
Perché riscrivi

http://operaez.net/mimetex/%5Cleft(1-%5Cfrac{y}{n}%5Cright)^n=%5Cleft(%5Cleft(1-%5Cfrac{y}{n}%5Cright)^{-%5Cfrac{n}{y}}%5Cright)^{-y}

e applichi un limite notevole che dovresti conoscere.

Fabietto206
03-01-2010, 16:22
Grazie mille!!!

Jarni
03-01-2010, 16:29
Grazie mille!!!

No: grazie 1000.
:asd:

misterx
03-01-2010, 17:18
ciao,
stavo studiando la seguente funzione: y = log(√(x+1) - x) e ne ho calcolato la derivata prima che è:


y' = 1 1
------------- * -------------------
√(x+1) -x 2*√(x+1) -1


per semplificarmi i conti in quanto vorrei determinare il valore della x per y' = 0 ho posto √(x+1) = t e di conseguenza x = t^2 -1

sostitendo ho che:

y' = 1 1
------------- * ---------
t - t^2 -1 2t -1



sempre che i conti siano corretti, come si procede ora ?
Ho provato a moltiplicare ma sinceramente il risultato non mi sembra per nulla corretto

grazie

Ziosilvio
03-01-2010, 18:09
ciao,
stavo studiando la seguente funzione: y = log(√(x+1) - x) e ne ho calcolato la derivata prima che è:


y' = 1 1
------------- * -------------------
√(x+1) -x 2*√(x+1) -1


per semplificarmi i conti in quanto vorrei determinare il valore della x per y' = 0 ho posto √(x+1) = t e di conseguenza x = t^2 -1

sostitendo ho che:

y' = 1 1
------------- * ---------
t - t^2 -1 2t -1



sempre che i conti siano corretti, come si procede ora ?
Ho provato a moltiplicare ma sinceramente il risultato non mi sembra per nulla corretto

grazie
Se http://operaez.net/mimetex/t=%5Csqrt{x+1} allora http://operaez.net/mimetex/x=t^2-1, quindi http://operaez.net/mimetex/%5Csqrt{x+1}-x=t-t^2+1.

Inoltre, la derivata di http://operaez.net/mimetex/%5Csqrt{x+1}-x è http://operaez.net/mimetex/%5Cfrac{1}{2%5Csqrt{x+1}}-1 e non http://operaez.net/mimetex/%5Cfrac{1}{2%5Csqrt{x+1}-1}.

misterx
03-01-2010, 18:46
Se http://operaez.net/mimetex/t=%5Csqrt{x+1} allora http://operaez.net/mimetex/x=t^2-1, quindi http://operaez.net/mimetex/%5Csqrt{x+1}-x=t-t^2+1.

Inoltre, la derivata di http://operaez.net/mimetex/%5Csqrt{x+1}-x è http://operaez.net/mimetex/%5Cfrac{1}{2%5Csqrt{x+1}}-1 e non http://operaez.net/mimetex/%5Cfrac{1}{2%5Csqrt{x+1}-1}.


ops, un errore di trascrizione nella derivata, anche a me viene come suggerisci :)

grazie per la correzione

Devil!
03-01-2010, 20:34
Ciao, come si fa a risolvere un'equazione differenziale di grado superiore al primo (quando compare anche la funzione di grado zero) tramite la creazione di un sistema di equazioni differenziali di primo grado?

Ad esempio, per risolvere y''(t) + y(t) = 0
creo il sistema seguente:
y'(t) = z(t)
z'(t) + y(t) = 0

In teoria integro la seconda equazione per ottenere z(t), sostituisco nella prima, integro nuovamente e ottengo y(t).
Il problema è: come integro la seconda equazione?

misterx
04-01-2010, 17:09
edit

Jarni
04-01-2010, 17:32
In teoria integro la seconda equazione per ottenere z(t), sostituisco nella prima, integro nuovamente e ottengo y(t).

Magari la teoria è sbagliata...:rolleyes:

Io, per risolvere questo tipo di equazioni, uso strumenti come autovalori, autovettori, matrici diagonali...

Devil!
04-01-2010, 18:57
Ciao, come si fa a risolvere un'equazione differenziale di grado superiore al primo (quando compare anche la funzione di grado zero) tramite la creazione di un sistema di equazioni differenziali di primo grado?

Ad esempio, per risolvere y''(t) + y(t) = 0

Risolto:

http://img231.imageshack.us/img231/1980/ode.png

misterx
08-01-2010, 17:38
ciao,
ho una domanda apparentemente banale nata durante un esercizio.
Ho la seguente funzione y=6x^2+5x+1 così fattorizzata (2x + 1)·(3x + 1) e mi chiedevo se vale sempre la seguente proprietà: se prendo la radice di 2x+1 e ne determino la x questa vale -1/2 e se prendo la radice di 3x+1 questa vale -1/3.
A questo punto se sostituisco alla x di 2x+1 il valore -1/3 ottengo -1/2 e viceversa: è un caso e se non lo è ha un senso ?

grazie

kierlo
09-01-2010, 12:22
il valore di x è uno scusa. Se metti x= -1/2 ti esce 0, idem con x=-1/3.
Oppure non ho capito la tua domanda.

misterx
09-01-2010, 14:30
il valore di x è uno scusa. Se metti x= -1/2 ti esce 0, idem con x=-1/3.
Oppure non ho capito la tua domanda.

ciao,
devi invertire!
La radice ottenuta dal fattore 2x+1=0 => x=-1/2 la devi sostituire in 3x+1=0 e ottieni
3*-1/2+1=-1/2
2*-1/3+1=-1/3

è un caso ?

Ziosilvio
09-01-2010, 18:16
ciao,
devi invertire!
La radice ottenuta dal fattore 2x+1=0 => x=-1/2 la devi sostituire in 3x+1=0 e ottieni
3*-1/2+1=-1/2
2*-1/3+1=-1/3

è un caso ?
Se moltiplichi 2 per -1/3 e aggiungi 1, ottieni +1/3.

misterx
09-01-2010, 19:18
vero, allora non c'entra nulla

ciao

easyand
10-01-2010, 16:42
ciao, sto avevendo qualche problema con questo esercizio

Trovare i punti stazionari delle funzioni:

a) ln(2x+y+2)-2x-y

b) xye (con e alla 4x quadro -5xy + y quadro)

misterx
10-01-2010, 17:32
ciao,
ho il seguente limite:

lim log1/2(1-x)
x-> +oo

lim log1/2(1-x)
x-> -oo

note: 1/2 è la base del logarimo

come lo si studia ?
Ho provato raccogliendo una x ma no mi convince il metodo

lim log1/2(x(1/x-1)) = -oo ?????
x-> +oo

grazie

Ziosilvio
10-01-2010, 18:41
ciao,
ho il seguente limite:

lim log1/2(1-x)
x-> +oo

lim log1/2(1-x)
x-> -oo

note: 1/2 è la base del logarimo

come lo si studia ?
Ho provato raccogliendo una x ma no mi convince il metodo

lim log1/2(x(1/x-1)) = -oo ?????
x-> +oo

grazie
Usa la legge del cambio di base:

http://operaez.net/mimetex/%5Clog_ab=%5Clog_ac%5Ccdot%5Clog_cb

Nel tuo caso, a=1/2 e c=e.

Ah: tieni d'occhio il segno dell'argomento del logaritmo.

diablo...aka...boss
11-01-2010, 15:06
Qualcuno di voi conosce un sito dove posso trovare esercizi svolti di analisi numerica? Nello specifico li cerco per metodo di newton-cotes(trapezi e simpson), metodo delle potenze
Cercando da google non riesco a trovare nulla che si avvicina a quello che mi serve :mc:

grazie.

DLz
11-01-2010, 16:38
Salve a tutti!Qualcuno mi sa dire come dovrei studiare la convergenza totale della serie di funzioni di termine generale
(sin(2x))^n/ln(n)^1/2
Di solio io la studio atteaverso uno studio di funzione del termine genrale con x variabile.Il problema è che la derivata di quel numeratore è: 2n(sin(2x))^n-1(cos(2x))
Come faccio a stabilire quando (sin(2x))^n-1 è maggiore o uguale a zero senza distinguere i casi n-1 pari e n-1 dispari? (n va da 2 a +infinito)
Grazie a tuti! :)

Edit: risolto,basta porre (sin(2x))^n = t e trattarla come serie di potenze :)

jestermask
11-01-2010, 19:42
Scusate non ricordo ne riesco a trovare il terzo punto del criterio di convergenza di Leibniz.

In giro trovo sempre che :
i)an>0, e che
ii)an+1<an

però mi ricordo che c'era un terzo punto per poter utilizzare leibniz nelle serie a segno alterno. Qual'era?

EDIT: dovrei aver risolto. Il terzo punto è semplicemente lim |an|=0 che corrisponde al criterio da soddisfare per cauchy per le serie in generale.

Ziosilvio
12-01-2010, 10:29
L'ennesimo termine della successione si scrive http://operaez.net/mimetex/a_{n} oppure a{n}.

Il criterio di Leibniz dice che:
Se i termini a{n} sono di segno alterno e se la successione dei valori assoluti dei termini a{n} è monotona decrescente infinitesima, allora la serie degli a{n} converge semplicemente.

jestermask
14-01-2010, 12:12
L'ennesimo termine della successione si scrive http://operaez.net/mimetex/a_{n} oppure a{n}.

Il criterio di Leibniz dice che:
Se i termini a{n} sono di segno alterno e se la successione dei valori assoluti dei termini a{n} è monotona decrescente infinitesima, allora la serie degli a{n} converge semplicemente.

Grazie ho le idee schiarite ora. Anche se ho un altra domanda (geometria).

Mi stò confondendo sulle verifiche delle applicazioni lineari.

Ad esempio f:R2--->R3 http://img684.imageshack.us/img684/3514/immaginenp.jpg (http://img684.imageshack.us/i/immaginenp.jpg/)

non riesco a trovare un modo per verificarne la linearità. Io mi ricordo che deve essere:
f(x+y) = f(x)+f(y) e f(kx) = kf(x) però mi impiccio sempre tantissimo.

Mi potreste spiegare senza magheggi o artifici come farlo (ora e per tutte le altre applicazioni che mi capitano)?

jacky guru
14-01-2010, 13:33
f (a , b) = (a-b, a+b, a-b)

Verifico che f(x+y)=f(x)+f(y)

Considero coppie (a,b) ; (c,d)

f(a,b) + f(c,d) = (a-b, a+b, a-b) + (c-d, c+d, c-d) = (a-b+c-d, a+b+c+d, a-b+c-d) = [(a+b)+(c-d), (a+b)+(c+d), (a-b)+(c-d)] = f[(a,b)+(c,d)]

Spero di aver fatto giusto :stordita:

In modo analogo, verifichi che f(ax) = a*f(x) (a costante)

jestermask
14-01-2010, 16:21
Si è come sono riuscito a fare io alla fine, solo che ogni volta me lo dimentico. E' una cosa che non mi entra dimostrare le uguaglianze che poi sono semplici tra l'altro:D

carcaroff
14-01-2010, 19:00
La probabilità costante che un pezzo prodotto da una data macchina sia difettoso è valutata al 12% . Sapendo che in magazzino sono presenti 1000 pezzi prodotti da quella macchina, quant'è la probabilità che tra essi vi siano più di 150 pezzi difettosi?

Aiuto.:mc: :confused:

Io ho uno 0,18%.............:mc:

Dani88
16-01-2010, 10:50
http://img695.imageshack.us/img695/3264/escom.png

http://img34.imageshack.us/img34/8418/codecogseqn.png

Dani88
16-01-2010, 11:00
La probabilità costante che un pezzo prodotto da una data macchina sia difettoso è valutata al 12% . Sapendo che in magazzino sono presenti 1000 pezzi prodotti da quella macchina, quant'è la probabilità che tra essi vi siano più di 150 pezzi difettosi?

Aiuto.:mc: :confused:

Io ho uno 0,18%.............:mc:
http://img695.imageshack.us/img695/8002/codecogseqn2.png

ermejo91
16-01-2010, 14:24
Ragazzi ho bisogno di un vostro aiuto, frequento il 5 anno del liceo scientifico, praticamente mi trovo con un problema del genere:

"Dato un triangolo isoscele di base a e lato l, considera i punti medi dei lati uguali e congiungili. Ottieni un nuovo triangolo isoscele, procedi nello stesso modo per ottenere figure simili.
1-Determina la successione delle misure delle aree dei triangolo.
2-Scrivi il termine n-esimo e calcola il limite"

Come si deve procedere per questo problema?? Le successioni le abbiamo appena iniziate, ma non le ho capite molto bene. Poi cosa significa scrivi il termine n-esimo???

Grazie.

carcaroff
16-01-2010, 16:56
http://img695.imageshack.us/img695/8002/codecogseqn2.png

Grazie.:)

jestermask
17-01-2010, 15:33
Scusatemi ma non riesco a fare l'esercizio:
http://img62.imageshack.us/img62/2958/immaginejo.th.jpg (http://img62.imageshack.us/i/immaginejo.jpg/)

Come faccio a verificare se è una base di autovettori? Poi il secondo punto mi chiede ti trovare la matrice Me(f) quindi non credo di poter fare la matrice con il polinomio caratteristico con autovalori-autovettori-autospazi e verificarlo da lì. O si?:mbe:

PacManZ
17-01-2010, 15:46
edit

Ziosilvio
17-01-2010, 19:36
Scusatemi ma non riesco a fare l'esercizio:
http://img62.imageshack.us/img62/2958/immaginejo.th.jpg (http://img62.imageshack.us/i/immaginejo.jpg/)

Come faccio a verificare se è una base di autovettori? Poi il secondo punto mi chiede ti trovare la matrice Me(f) quindi non credo di poter fare la matrice con il polinomio caratteristico con autovalori-autovettori-autospazi e verificarlo da lì. O si?:mbe:
Base: basta verificare che i tre vettori sono linearmente indipendenti.
Di autovettori: devi calcolare la matrice A associata ad f, e verificare che manda ciascun vettore dell'insieme proposto in un suo multiplo.

misterx
17-01-2010, 19:42
ciao,
come si determina la derivata prima di: y = |x-1|^3
mi verrebbe da scrivere: y' = 3(x-1)^2 = (3x -3)^2 = 9x^2 - 9x + 9
però non conoscendone i motivi vi chiedo lumi che è meglio; di sicuro c'è di mezzo un punto angoloso come di consueto si ha nel caso del valore assoluto.

jacky guru
17-01-2010, 19:50
ciao,
come si determina la derivata prima di: y = |x-1|^3
mi verrebbe da scrivere: y' = 3(x-1)^2 = (3x -3)^2 = 9x^2 - 9x + 9
però non conoscendone i motivi vi chiedo lumi che è meglio; di sicuro c'è di mezzo un punto angoloso come di consueto si ha nel caso del valore assoluto.

Ciao, che io sappia esistono due maniere
1) Dividi la funzione di partenza in due "parti", "togliendo" il valore assoluto a seconda se x è maggiore o minore di 1, dunque derivi ciascuna delle due funzioni;
2) Derivi immediatamente, sapendo che la derivata del valore assoluto è del tipo "x/|x|", ovvero la funzione segno sign(x).

Ovviamente devi verificare la presenza del punto angoloso in x=1.

jestermask
17-01-2010, 19:52
Base: basta verificare che i tre vettori sono linearmente indipendenti.
Di autovettori: devi calcolare la matrice A associata ad f, e verificare che manda ciascun vettore dell'insieme proposto in un suo multiplo.

Ah porca miseria la matrice la costruisco su base A!! Io la facevo sulla base canonica ed infatti non veniva...pero' ho rifatto al volo i conti con Ma(f) ed ho

2 0 1
0 2 1
0 0 1

che sono proprio {2e1; 2e2; e3}.

Grazie, finalmente posso dormire tranquillo e soddisfatto:D

jacky guru
17-01-2010, 20:02
Ah porca miseria la matrice la costruisco su base A!! Io la facevo sulla base canonica ed infatti non veniva...pero' ho rifatto al volo i conti con Ma(f) ed ho

2 0 1
0 2 1
0 0 1

che sono proprio {2e1; 2e2; e3}.

Grazie, finalmente posso dormire tranquillo e soddisfatto:D

Esame di Geometria? :) Io ce l'ho questo 25 :stordita:

jestermask
17-01-2010, 20:05
Esame di Geometria? :) Io ce l'ho questo 25 :stordita:

io il 30 (sabato:rolleyes: )

kwb
17-01-2010, 20:13
ciao,
come si determina la derivata prima di: y = |x-1|^3
mi verrebbe da scrivere: y' = 3(x-1)^2 = (3x -3)^2 = 9x^2 - 9x + 9
però non conoscendone i motivi vi chiedo lumi che è meglio; di sicuro c'è di mezzo un punto angoloso come di consueto si ha nel caso del valore assoluto.

Dividi la funzione secondo i casi in cui x sia negativo e positivo.
Poi derivi separatamente.

misterx
18-01-2010, 05:57
Ciao, che io sappia esistono due maniere
1) Dividi la funzione di partenza in due "parti", "togliendo" il valore assoluto a seconda se x è maggiore o minore di 1, dunque derivi ciascuna delle due funzioni;
2) Derivi immediatamente, sapendo che la derivata del valore assoluto è del tipo "x/|x|", ovvero la funzione segno sign(x).

Ovviamente devi verificare la presenza del punto angoloso in x=1.

ciao,
la funzione sign ci è stata solo accennata ma non spiegata. Intendi che devo derivare:
(x-1)^3
-(x-1)^3
e poi studiarne il limite per x -> 1 per scoprire il punto angoloso ?

diablo...aka...boss
18-01-2010, 15:34
Domanda sugli autovalori/autovettori:

come si trovano gli autovalori mi è chiaro, per trovare gli autovettori devo sostituire il valore di lambda1 ad esempio nella matrice A-I*lambda , così da trovare gli autovettori per l'autovalore lambda1 . è corretto il procedimento che faccio?

grazie.

jacky guru
18-01-2010, 16:42
ciao,
la funzione sign ci è stata solo accennata ma non spiegata. Intendi che devo derivare:
(x-1)^3
-(x-1)^3
e poi studiarne il limite per x -> 1 per scoprire il punto angoloso ?
Yess!
Domanda sugli autovalori/autovettori:

come si trovano gli autovalori mi è chiaro, per trovare gli autovettori devo sostituire il valore di lambda1 ad esempio nella matrice A-I*lambda , così da trovare gli autovettori per l'autovalore lambda1 . è corretto il procedimento che faccio?

grazie.
Esatto, risolvi il sistema omogeneo (A-TI)X=0 , dove T è ogni radice del polinomio caratteristico della matrice A (ovvero ogni autovalore) ed I la matrice identica.
Ovviamente il sistema avrà perlomeno "infinito alla uno" soluzioni, per essere congruente alla definizione di autovettore ;)

Ziosilvio
18-01-2010, 17:43
Domanda sugli autovalori/autovettori:

come si trovano gli autovalori mi è chiaro, per trovare gli autovettori devo sostituire il valore di lambda1 ad esempio nella matrice A-I*lambda , così da trovare gli autovettori per l'autovalore lambda1 . è corretto il procedimento che faccio?

grazie.
Se intendi "per ogni autovalore t trovo le soluzioni del sistema (A-tI)x=0", allora sì.

diablo...aka...boss
18-01-2010, 18:57
Se intendi "per ogni autovalore t trovo le soluzioni del sistema (A-tI)x=0", allora sì.

intendo che sostituendo lambda1 nella matrice trovo gli autovalori per lambda1, allora è quello che indendo io. poi ovviamente dovrò fare la stessa cosa per gli altri lambda che ho.

Dani88
19-01-2010, 09:30
Esercizio postato in precedenza risolto :D :D
Ora avrei bisogno di un altro aiuto, sui processi aleatori però...:mc: :mc:
:help: :help: :help:
http://img163.imageshack.us/img163/6177/codecogseqn3.png

misterx
19-01-2010, 16:56
ciao,
ho la seguente funzione:

y = x log x
----------
x^2 - 4

ho determinato la sua derivata prima


2 2
(x + 4)·LN(x) - x + 4
y' = - -------------------------
2 2
(x - 4)



ora pongo la y' = 0 per determinare le radici che la annullano ma come si determinano ?

Il denominatore è sempre positivo e quindi posso evitare di farci su i conti ma il numeratore ?

grazie

kierlo
19-01-2010, 18:48
Io confronterei le due funzioni:

Tralasciando il meno davanti alla frazione per semplicità trovi per quali valori

log x > (x^-4)/(x^2+4)= 1-8/(x^2+4)

Così vedi per quali valori é positiva o negativa e anche gli zeri. In caso la porti in forma esponenziale che magari è + comoda da maneggiare, altrimenti non saprei, ora come ora

Devil!
19-01-2010, 19:11
ciao,
ho la seguente funzione:

y = x log x
----------
x^2 - 4

ho determinato la sua derivata prima


2 2
(x + 4)·LN(x) - x + 4
y' = - -------------------------
2 2
(x - 4)



ora pongo la y' = 0 per determinare le radici che la annullano ma come si determinano ?

Il denominatore è sempre positivo e quindi posso evitare di farci su i conti ma il numeratore ?

grazie

a me la tua y' non torna; mi viene:

[(ln(x)+1)*(x²-4)-2x²ln(x)]/(x²-4)²

misterx
19-01-2010, 19:29
Io confronterei le due funzioni:

Tralasciando il meno davanti alla frazione per semplicità trovi per quali valori

log x > (x^-4)/(x^2+4)= 1-8/(x^2+4)

Così vedi per quali valori é positiva o negativa e anche gli zeri. In caso la porti in forma esponenziale che magari è + comoda da maneggiare, altrimenti non saprei, ora come ora

ah ecco l'idea, non ci sarei mai arrivato; grazie 1000

misterx
19-01-2010, 19:32
a me la tua y' non torna; mi viene:

[(ln(x)+1)*(x²-4)-2x²ln(x)]/(x²-4)²


io ho applcato:

f(x)g(x)/h(x) = R(x)/H(x)

R'(x)*H(x)-R(x)*H'(x)
--------------------
H(x)²

dove R'(x)=f'(x)*g(x)+f(x)*g'(x)

comunque l'ho poi verificata con derive

misterx
20-01-2010, 20:51
ciao,
qualcuno mi dice come si determina la radice da equazioni di questo tipo?

e^x(-2-x) - 1 = 0

grazie

flapane
20-01-2010, 21:05
Quand'è che l'esponenziale è uguale a 1 (portando il -1 a destra dell'uguaglianza) ? --->
---> Quand'è che x*(-2-x)=0 ? -->
---> x[i]=[x1,x2...]

misterx
20-01-2010, 21:09
Quand'è che l'esponenziale è uguale a 1 (portando il -1 a destra dell'uguaglianza) ? --->
---> Quand'è che x*(-2-x)=0 ? -->
---> x[i]=[x1,x2...]

ciao,
che e^x = 1 quando x=0 ci ero arrivato ma non ho capito quel x*(-2-x)

flapane
20-01-2010, 21:12
fai in modo di vedere quel x*(-2-x) come la tua x generica, e ponilo uguale a 0.

Quiksilver
21-01-2010, 14:59
Ciao a tutti..

mi spiegate questa derivata?

D(3 + x + senx) = 1 + cosx

grazie

flapane
21-01-2010, 15:09
http://www.google.it/search?source=ig&hl=it&rlz=&=&q=tavole+derivate&btnG=Cerca+con+Google&meta=lr%3D&aq=f&oq=

Puoi svolgere singolarmente le derivate di quegli addendi.

Quiksilver
21-01-2010, 15:17
http://www.google.it/search?source=ig&hl=it&rlz=&=&q=tavole+derivate&btnG=Cerca+con+Google&meta=lr%3D&aq=f&oq=

Puoi svolgere singolarmente le derivate di quegli addendi.

grazie

gugoXX
21-01-2010, 16:52
Esistono per le serie delle regole di calcolo che non siano solo sperare di riuscire a ridurre a serie note?

Regole generali un po' come per gli integrali indefiniti intendo.

Es: La sommatoria del prodotto f(t)*g(t)

Ziosilvio
21-01-2010, 17:18
Esistono per le serie delle regole di calcolo che non siano solo sperare di riuscire a ridurre a serie note?

Regole generali un po' come per gli integrali indefiniti intendo.

Es: La sommatoria del prodotto f(t)*g(t)
Per il prodotto di due serie ricordiamo che vale questa regola:
Se http://operaez.net/mimetex/S=%5Csum_{n=0}^{%5Cinfty}a_n e http://operaez.net/mimetex/T=%5Csum_{n=0}^{%5Cinfty}b_n allora http://operaez.net/mimetex/S%5Ccdot{T}=%5Csum_{n=0}^{%5Cinfty}c_n con http://operaez.net/mimetex/c_n=%5Csum_{k=0}^{n}a_kb_{n-k}.
Un teorema di Abel dice che, se la serie prodotto converge, allora converge al prodotto delle somme delle serie fattore.
Il guaio è che, se S e T convergono, non è assicurato che converga anche S*T! Un esempio (Rudin, Principles of Mathematical Analysis, Example 3.49): http://operaez.net/mimetex/a_n=%5Cfrac{(-1)^n}{%5Csqrt{n+1}}.
Esiste però il teorema di Mertens: se entrambe le serie convergono e se una delle due converge assolutamente, allora la serie prodotto converge semplicemente.

misterx
21-01-2010, 18:49
scusate edit

misterx
21-01-2010, 18:56
fai in modo di vedere quel x*(-2-x) come la tua x generica, e ponilo uguale a 0.

forse ho capito, grazie

matrix866
21-01-2010, 21:09
Qualche buon anima mi può spiegare come risolvere il seguente integrale doppio? Con che metodo?


http://img14.imageshack.us/img14/4918/doppioq.jpg (http://img14.imageshack.us/i/doppioq.jpg/)

Gli estremi di integrazione che ho trovato io sono:

0 < x < log(2)

1/2 < y < 2

Mi basta solo il metodo da applicare non serve lo svolgimento completo. Non vi chiedo tanto :D

Grazie a tutti!!!

gugoXX
22-01-2010, 08:41
Per il prodotto di due serie ricordiamo che vale questa regola:
Se http://operaez.net/mimetex/S=%5Csum_{n=0}^{%5Cinfty}a_n e http://operaez.net/mimetex/T=%5Csum_{n=0}^{%5Cinfty}b_n allora http://operaez.net/mimetex/S%5Ccdot{T}=%5Csum_{n=0}^{%5Cinfty}c_n con http://operaez.net/mimetex/c_n=%5Csum_{k=0}^{n}a_kb_{n-k}.
Un teorema di Abel dice che, se la serie prodotto converge, allora converge al prodotto delle somme delle serie fattore.
Il guaio è che, se S e T convergono, non è assicurato che converga anche S*T! Un esempio (Rudin, Principles of Mathematical Analysis, Example 3.49): http://operaez.net/mimetex/a_n=%5Cfrac{(-1)^n}{%5Csqrt{n+1}}.
Esiste però il teorema di Mertens: se entrambe le serie convergono e se una delle due converge assolutamente, allora la serie prodotto converge semplicemente.

Grazie. L'idea si avvicina molto a cosa avrei bisogno.
Purtroppo pero' devo calcolare la serie del prodotto di due funzioni fino ad infinito, e non il prodotto di due serie (fino ad infinito). Non riesco a vedere se riesco ad applicare.

http://operaez.net/mimetex/S=%5Csum_%7Bn=0%7D%5E%7B%5Cinfty%7Da_nb_n

Ziosilvio
22-01-2010, 09:53
Grazie. L'idea si avvicina molto a cosa avrei bisogno.
Purtroppo pero' devo calcolare la serie del prodotto di due funzioni fino ad infinito, e non il prodotto di due serie (fino ad infinito). Non riesco a vedere se riesco ad applicare.

http://operaez.net/mimetex/S=%5Csum_%7Bn=0%7D%5E%7B%5Cinfty%7Da_nb_n
Se le funzioni sono analitiche, allora direi che puoi adoperare il prodotto dei loro sviluppi in serie di potenze.

gugoXX
22-01-2010, 10:21
Se le funzioni sono analitiche, allora direi che puoi adoperare il prodotto dei loro sviluppi in serie di potenze.

Si', sono analitiche.
Vediamo.

bilottifra
22-01-2010, 17:29
ciao a tutti sono uovo di questa utilissima discussione, non riesco a risolvere questo problema spero che mi possiate aiutare. Grazie


Ad assistere a una partita di calcio ci sono 60000 paganti. il prezzo del bilgietto della tribuna è di 40 euro distinti 25 e curva 15. Sapendo che l'incasso è di 1.260.000 e che gli spettatori nei distinti sono il doppio di quelli in tribuna, calcolare quanti sono gli spettatori paganti in tribuna in distinti e in curva...

R: 8000, 1600 36000

Ziosilvio
22-01-2010, 17:39
Ad assistere a una partita di calcio ci sono 60000 paganti. il prezzo del bilgietto della tribuna è di 40 euro distinti 25 e curva 15. Sapendo che l'incasso è di 1.260.000 e che gli spettatori nei distinti sono il doppio di quelli in tribuna, calcolare quanti sono gli spettatori paganti in tribuna in distinti e in curva...

R: 8000, 1600 36000
Chiama t il numero di spettatori in tribuna, d quello degli spettatori in distinti, c quello degli spettatori in curva.
I dati del problema ti dicono che:

t + d + c = 60000
40*t + 25*d + 15*c = 1260000
2*t - d = 0

Questo è un sistema di tre equazioni lineari nelle tre incognite t, d, c; che sai sicuramente risolvere con i metodi che ti hanno insegnato.

bilottifra
22-01-2010, 17:48
io sto al secondo anno del classico non l'ho fatto sto sistema :(

MaxArt
22-01-2010, 17:54
io sto al secondo anno del classico non l'ho fatto sto sistema :(Allora proviamo come equazione (che in realtà sarà la risolvente del sistema che ha scritto Ziosilvio - la vedrai presto).
Se x è il numero di spettatori in tribuna, 2x è quello sui distinti, mentre quelli in curva sono i rimanenti, quindi ...
Imposti l'equazione:
x*40+2x*25+(...)*15 = 1260000
Nei puntini mettici gli spettatori della curva. Questa la sai fare.

Ziosilvio
22-01-2010, 18:00
io sto al secondo anno del classico non l'ho fatto sto sistema :(
Però al secondo anno del classico dovresti aver visto i sistemi di due equazioni lineari in due incognite.
Qui puoi ricondurti a uno di questi se osservi che la terza equazione ti permette di riscrivere le d come se fossero t. Infatti, se 2*d-t=0, o che è lo stesso t = 2*d, allora puoi scrivere 2*t al posto di d nelle altre due equazioni.
Quindi il tuo sistema diventa:

t + (2*t) + c = 60000
40*t + 25*(2*t) + 15*c = 1260000
d = 2*t

ossia, svolgendo,

3*t + c = 60000
90*t + 15*c = 1260000
d = 2*t

Le prime due righe formano un sistema di due equazioni lineari delle due incognite t e c, che sai risolvere.
Trovato t, trovi subito d per mezzo della terza equazione.

Nota: buona parte del lavoro di un matematico, consiste nel riportare problemi nuovi a problemi noti...

gugoXX
22-01-2010, 18:05
Se le funzioni sono analitiche, allora direi che puoi adoperare il prodotto dei loro sviluppi in serie di potenze.

No. Non sono analitiche mi sa.
Una delle 2 e' il logaritmo, che a 0 va a -inf.
Quindi non posso neppure scriverla la MacLaurin in 0 (che peraltro non mi interessa)
e se anche mi spostassi con Taylor, non raggiungerei la convergenza fino ad infinito.

e anche la radice e' pessima.

http://operaez.net/mimetex/%5Csum_%7Bn=1%7D%5E%7B%5Cinfty%7D%5Cfrac%7Bx%5Cln%7B%28n%29%7D%7D%7B%5Csqrt%7Bn%7D%7D=0

Alla fine devo trovare le X che mi soddisfano l'equazione.
Come confronto il logaritmo con la radice? La radice e' sempre maggiore del logaritmo in ogni intervallo? (Direi di si)
X sara' quindi sempre e solo 0?


Altra domanda correlata, ma l'integrale di una serie, e' uguale alla serie degli integrali? Quand'e' che posso invertire i due simboli?
Se la funzione e' continua, derivabile e/o integrabile a piacere, con derivate e integrali continui in tutto l'intervallo di interesse (per me 1 -> inf), posso scambiare i segni?

Ziosilvio
22-01-2010, 20:59
No. Non sono analitiche mi sa.
Una delle 2 e' il logaritmo, che a 0 va a -inf.
Quindi non posso neppure scriverla la MacLaurin in 0 (che peraltro non mi interessa)
e se anche mi spostassi con Taylor, non raggiungerei la convergenza fino ad infinito.

e anche la radice e' pessima.

http://operaez.net/mimetex/%5Csum_%7Bn=1%7D%5E%7B%5Cinfty%7D%5Cfrac%7Bx%5Cln%7B%28n%29%7D%7D%7B%5Csqrt%7Bn%7D%7D=0

Alla fine devo trovare le X che mi soddisfano l'equazione.
Come confronto il logaritmo con la radice? La radice e' sempre maggiore del logaritmo in ogni intervallo? (Direi di si)
X sara' quindi sempre e solo 0?


Altra domanda correlata, ma l'integrale di una serie, e' uguale alla serie degli integrali? Quand'e' che posso invertire i due simboli?
Se la funzione e' continua, derivabile e/o integrabile a piacere, con derivate e integrali continui in tutto l'intervallo di interesse (per me 1 -> inf), posso scambiare i segni?
Aspetta, aspetta: quindi la somma che devi calcolare è

http://operaez.net/mimetex/%5Csum_{n=1}^{%5Cinfty}%5Cfrac{x%5Cln{n}}{%5Csqrt{n}}

Se è così, allora converge per x=0 e diverge per x<>0. ESERCIZIO: dimostrare.

Per quanto riguarda la seconda domanda, è vero che, se la serie

http://operaez.net/mimetex/%5Csum_{n=1}^{%5Cinfty}f_n(x)

converge uniformemente nell'intervallo di integrazione, allora

http://operaez.net/mimetex/%5Cint%5Cleft(%5Csum_{n=1}^{%5Cinfty}f_n(x)%5Cright)dx=%5Csum_{n=1}^{%5Cinfty}%5Cint{f_n(x)dx}

In generale, però, non si può scambiare l'integrale della serie con la serie degli integrali.

gugoXX
22-01-2010, 22:41
Aspetta, aspetta: quindi la somma che devi calcolare è

http://operaez.net/mimetex/%5Csum_{n=1}^{%5Cinfty}%5Cfrac{x%5Cln{n}}{%5Csqrt{n}}

Se è così, allora converge per x=0 e diverge per x<>0. ESERCIZIO: dimostrare.

Per quanto riguarda la seconda domanda, è vero che, se la serie

http://operaez.net/mimetex/%5Csum_{n=1}^{%5Cinfty}f_n(x)

converge uniformemente nell'intervallo di integrazione, allora

http://operaez.net/mimetex/%5Cint%5Cleft(%5Csum_{n=1}^{%5Cinfty}f_n(x)%5Cright)dx=%5Csum_{n=1}^{%5Cinfty}%5Cint{f_n(x)dx}

In generale, però, non si può scambiare l'integrale della serie con la serie degli integrali.

Direi per confronto.
Poiche' ln(n) e' sempre >1 e sqrt(n) e' sempre <n, allora ne consegue che ln(n)/sqrt(n) e' sempre maggiore di 1/n
e poiche' la sommatoria di 1/n diverge, allora diverge anche la ln(n)/sqrt(n)
A meno che x valga 0 ovviamente, che converge sempre e la somma e' appunto 0.


Ma questo e' solo un pezzo. Devo fare tanto di piu' e tanto piu' brutto.
Ma c'e' da dire che sono sicuro di avere successioni che convergono sempre, anche se non saprei dire se assolutamente, in tutto l'intervallo.
Immagina una sommatoria come quella di prima, e che io non mi sia accorto che e' banale, e che non diverga sempre e che abbia qualche soluzione per qualche x

http://operaez.net/mimetex/%5Csum_{n=1}^{%5Cinfty}%5Cfrac{x%5Cln{n}}{%5Csqrt{n}}

L'idea dell'integrale della sommatoria e dell'inversione dei simboli e' questa: Non so se riesco a trovare una forma chiusa per la sommatoria, ma penso che ci riuscirei per la derivata della fuinzione della successione.
Posso calcolare la sommatoria della derivata, ottengo una funzione in X, che invece che eguagliare a 0 mi metterei a studiare cercando i massimi e i minimi?
Dovrei ottenere le stesse x che soddisfavano il criterio di partenza, o no?

Edit: No. Non posso derivare su N e poi calcolare i massimi e i minimi secondo X, non ha senso. Niente, cancello.

ShadowMan
23-01-2010, 10:59
edit: cannato io XD

Jarni
24-01-2010, 16:47
Mi son inchiodato su questo esercizietto.

(1/4)^(-x^2) >= 2 che posso riscrivere
2^(2x^2) => 2^1 <=> 2x^2 => 1 <=> x >= +- sqrt(1/2)

Nella soluzione dice x €(-infinito; -1/2*sqrt(2)] U [1/2*sqrt(2) ; infinito) :mbe:

Cosa ho sbagliato ? :(

Se hai questo:

2x^2 >= 1

la soluzione non puoi scriverla così:

x >= +- sqrt(1/2)

Che significa? x maggiore o uguale ad un numero positivo oppure negativo? Non ha molto senso...

Lo sai come si risolvono le disequazioni di II grado?

ShadowMan
24-01-2010, 17:45
Penso di aver modificato il mio messaggio un istante dopo che tu rispondessi perché mi ero reso conto dell'errore :fagiano:

Jarni
24-01-2010, 17:48
Penso di aver modificato il mio messaggio un istante dopo che tu rispondessi perché mi ero reso conto dell'errore :fagiano:

La mano è più veloce dell'occhio...

misterx
24-01-2010, 20:02
ciao,
mi chiedevo quali strumenti si hanno a disposizione per capire se una funzione è invertibile.
So che deve possedere le due seguenti proprietà:
- iniettiva
- suriettiva

Non mi è chiaro pero come si deve fare per scoprire se una determinata funzione è biiettiva o meno: si va per ragionamento oppure esistono dei metodi meccanici ?

Esempio: y = log ( |x| + 1 )

a occhio il logaritmo mi sembra una funzione suriettiva in quanto copre tutto R(sto pensando all'asse y) e per ogni numero associato a x ho un solo numero associato a y e quindi mi pare risulti anche iniettiva.

La sua inversa, quella del logaritmo è esponenziale e dovremmo essere a posto però, c'è il modulo a complicare le cose e a occhio non mi pare sia nè suriettivo nè iniettivo, e quindi credo che il modulo "rovini" li biiettività del logaritmo: è così ?

Supponendo che abbia indovinato basandomi sul ragionamento, quando si ha a che fare con funzioni più complesse come si stabilisce se è biiettiva una funzione e quindi invertibile ?


grazie

Jarni
25-01-2010, 00:32
ciao,
mi chiedevo quali strumenti si hanno a disposizione per capire se una funzione è invertibile.
So che deve possedere le due seguenti proprietà:
- iniettiva
- suriettiva

Non mi è chiaro pero come si deve fare per scoprire se una determinata funzione è biiettiva o meno: si va per ragionamento oppure esistono dei metodi meccanici ?

Esempio: y = log ( |x| + 1 )

a occhio il logaritmo mi sembra una funzione suriettiva in quanto copre tutto R(sto pensando all'asse y) e per ogni numero associato a x ho un solo numero associato a y e quindi mi pare risulti anche iniettiva.

La sua inversa, quella del logaritmo è esponenziale e dovremmo essere a posto però, c'è il modulo a complicare le cose e a occhio non mi pare sia nè suriettivo nè iniettivo, e quindi credo che il modulo "rovini" li biiettività del logaritmo: è così ?

Supponendo che abbia indovinato basandomi sul ragionamento, quando si ha a che fare con funzioni più complesse come si stabilisce se è biiettiva una funzione e quindi invertibile ?


grazie

Se hai un minimo o un massimo in un punto, chiaramente non puoi invertire la funzione.:D

misterx
25-01-2010, 05:54
Se hai un minimo o un massimo in un punto, chiaramente non puoi invertire la funzione.:D

quindi se esiste la derivata prima e si annulla non è invertibile ?

jacky guru
25-01-2010, 09:04
Ciao ragazzi, come verifico la complanarità di TRE rette tra loro parallele? Prodotto misto uguale a zero? Mi sembra sin troppo facile... :stordita:

gugoXX
25-01-2010, 09:06
Ciao ragazzi, come verifico la complanarità di TRE rette tra loro parallele? Prodotto misto uguale a zero? Mi sembra sin troppo facile... :stordita:

Se le norme di 2 qualsiasi prodotti vettoriali sono uguali.

gugoXX
25-01-2010, 09:09
quindi se esiste la derivata prima e si annulla non è invertibile ?

Direi di no.
Y=X^3 ha un punto con derivata nulla, ma si puo' invertire.

E non basta nemmeno controllare la derivata seconda in tale punto.
Y=X^4 ha sia derivata prima con derivata seconda nulla nello stesso punto, ma non si puo' invertire...

Dani88
25-01-2010, 09:12
Avendo N oggetti, le possibili coppie, triplette, ecc... come le calcolo?
Io ho pensato: facendo il binomiale tra N e 3 (ad es) ottengo le triplette NON ordinate.
Se ora moltiplico tutto per 3! ottengo quelle ordinate...giusto o ca***a immane? :D :D

ad esempio con 52 carte, estraendone 2 di fila (senza rimetterle nel mazzo) posso ottenere
(52 2) =1326 coppie non ordinate
52! * (52 2) =~ 1,06*10^71 coppie ordinate (possibile??)

jacky guru
25-01-2010, 09:25
Se le norme di 2 qualsiasi prodotti vettoriali sono uguali.

Ciao,
grazie per la risposta!

Ma se i tre vettori sono comunque tra loro paralleli... qualunque prodotto vettoriale io faccia sarà sempre il vettore nullo (di norma zero)! O sbaglio? :confused:

gugoXX
25-01-2010, 10:24
Ciao,
grazie per la risposta!

Ma se i tre vettori sono comunque tra loro paralleli... qualunque prodotto vettoriale io faccia sarà sempre il vettore nullo (di norma zero)! O sbaglio? :confused:

Hai ragione, avevo perso il "parallele", e pensavo dovessi verificare solo la complanarita'.
Allora si', il prodotto misto uguale a 0 dovrebbe essere corretto, ma devi anche verificare che le rette siano tra loro parallele (a meno che non sia un dato di ipotesi)

Jarni
25-01-2010, 12:35
quindi se esiste la derivata prima e si annulla non è invertibile ?

L'annullamento della derivata prima non basta per trovare un estremo locale...

jacky guru
25-01-2010, 17:55
Direi di no.
Y=X^3 ha un punto con derivata nulla, ma si puo' invertire.

E non basta nemmeno controllare la derivata seconda in tale punto.
Y=X^4 ha sia derivata prima con derivata seconda nulla nello stesso punto, ma non si puo' invertire...

Beh, nel primo caso infatti la derivata in x=0 è nulla, tuttavia NON è un estremo relativo ma un punto di flesso. Credo vada verificata la presenza di estremi relativi applicando il relativo teorema agli zeri della derivata (considerando la derivata seconda, vedendo se tale punto rende la derivata seconda maggiore o minore di zero, se uguale a zero passare alla derivata terza, ecc....).

Hai ragione, avevo perso il "parallele", e pensavo dovessi verificare solo la complanarita'.
Allora si', il prodotto misto uguale a 0 dovrebbe essere corretto, ma devi anche verificare che le rette siano tra loro parallele (a meno che non sia un dato di ipotesi)

Mmh provo a rispondermi nuovamente: il prodotto misto non credo aiuti poichè abbiamo verificato che comunque i vettori // alle rette sono tra loro //, ma in tal caso il prodotto misto non credo assicuri la complanarità. Credo, a questo punto, che basti trovare il piano passante per due di queste rette e poi vedere se un punto qualunque della terza retta appartiene anch'esso a tale piano: in tal caso, le tre rette giacciono sullo stesso piano sicuramente.

______________________


Una domanda (presente sull'esame di geometria fatto oggi :stordita: ): l'equazione 2xy - z^2 = 0 è relativa ad una superficie conica, vero? :)



.

gugoXX
25-01-2010, 18:38
Mmh provo a rispondermi nuovamente: il prodotto misto non credo aiuti poichè abbiamo verificato che comunque i vettori // alle rette sono tra loro //, ma in tal caso il prodotto misto non credo assicuri la complanarità. Credo, a questo punto, che basti trovare il piano passante per due di queste rette e poi vedere se un punto qualunque della terza retta appartiene anch'esso a tale piano: in tal caso, le tre rette giacciono sullo stesso piano sicuramente.

Gia'. Hai ragione, il prodotto misto non basta.
Ma mi sa che non basta neppure il tuo metodo. Una qualsiasi retta sghemba alle 2 potrebbe intercettare il piano in un punto.
Due punti e dovresti essere a posto.


Ma ho pensato a questo, che dovrebbe essere altrettanto semplice.
Per verificare se le 3 rette sono parallele e complanari,
prendo un punto casuale da ciascuna retta. Ho 3 punti e calcolo il prodotto vettoriale, ottengo il vettore del piano su cui giacciono (se ottengo 0 sono stato davvero tanto, tantissimo sfortunato. Ne prendo un altro a caso da una qualsiasi retta e ricomincio).
Poi tengo fermo il punto sulla retta 1, prendo altri 2 punti casuali sulle altre 2 rette e rifo, ottenendo un altro vettore.
Se i due vettori hanno la stesso versore (direzione a parte), allora ho vinto, sono veramente sullo stesso piano. altrimenti sono su piani diversi.

Una volta appurato che sono sullo stesso piano, allora posso fare lo scalare o il vettoriale delle rette a 2 a 2, e se sono sempre tutti 0, allora sono anche parallele.

misterx
25-01-2010, 22:23
Direi di no.
Y=X^3 ha un punto con derivata nulla, ma si puo' invertire.

E non basta nemmeno controllare la derivata seconda in tale punto.
Y=X^4 ha sia derivata prima con derivata seconda nulla nello stesso punto, ma non si puo' invertire...

ok grazie, quindi biunivoca e monotona

jacky guru
25-01-2010, 23:58
Ma mi sa che non basta neppure il tuo metodo. Una qualsiasi retta sghemba alle 2 potrebbe intercettare il piano in un punto.

No, credo basti perchè ho già verificato che i tre vettori ad esse paralleli sono tra loro paralleli, dunque le tre rette non potranno mai essere sghembe! :)

diablo...aka...boss
26-01-2010, 18:39
dovrei chiedere qualche info su probabilità e statistica, posso farlo quì, c'è un 3d specifico che mi è sfuggito, o ne devo aprire uno nuovo ?

ShadowMan
26-01-2010, 18:55
Come passo da qui
http://img43.imageshack.us/img43/4963/49876892.jpg
a qui :mbe: :confused: :mbe:
http://img515.imageshack.us/img515/2966/97609736.jpg

per x-> -oo

Jarni
26-01-2010, 19:04
Come passo da qui
http://img43.imageshack.us/img43/4963/49876892.jpg
a qui :mbe: :confused: :mbe:
http://img515.imageshack.us/img515/2966/97609736.jpg

per x-> -oo

Non si capisce bene qual'è l'argomento del logaritmo nella seconda espressione.

ShadowMan
26-01-2010, 19:08
Non si capisce bene qual'è l'argomento del logaritmo nella seconda espressione.

Non essendoci le parentesi tra ln 2 e (1+o(1)) penso che l'argomento sia tutto. Cioè 2 che moltiplica (1+o(1)) :stordita:

kierlo
26-01-2010, 19:41
Basta semplicemente mettere in evidenza x ln 2:
così ti esce [x ln2](1 -x^(1/3)/(xln2)]
Ovviamente la radice quadrata è un o piccolo di x per x-->+ infinito, da ciò l'espressione.

misterx
27-01-2010, 12:22
ciao,
ho il seguente esercizio:

determinare a,b appartenenti R :

ax+1 , |x-1| < 2
f(x) =
bcos(pigreco/3x) , |x-1| >= 2


dopo aver svolto la disequazione ho trovato che:
per |x-1| < 2 la funzione si trova nell'intervallo (-1,3)
per |x-1| >= 2 la funzione invece si trova nell'intervallo (x <= -1) v (x >= 3)

fatto questo ho scritto:
-a + x < y < 3a + x immagine della y per quanto riguarda il parametro (a)
y <= bcos(-pigreco/3) v y >= bcos(pigreco) per quanto riguarda il parametro (b)

è corretto ?

Jarni
27-01-2010, 13:38
ciao,
ho il seguente esercizio:

determinare a,b appartenenti R :

ax+1 , |x-1| < 2
f(x) =
bcos(pigreco/3x) , |x-1| >= 2


dopo aver svolto la disequazione ho trovato che:
per |x-1| < 2 la funzione si trova nell'intervallo (-1,3)
per |x-1| >= 2 la funzione invece si trova nell'intervallo (x <= -1) v (x >= 3)

OK.

fatto questo ho scritto:
-a + x < y < 3a + x immagine della y per quanto riguarda il parametro (a)
y <= bcos(-pigreco/3) v y >= bcos(pigreco) per quanto riguarda il parametro (b)

è corretto ?

Qui mi sono perso. Ma l'esercizio che chiede?
"Determinare a e b", va bene, ma "affinché" succeda cosa?
Servirà una qualche condizione che deve essere rispettata dai due parametri...

misterx
27-01-2010, 13:58
ciao,
chiede di determinare a,b e per tali valori l'insieme delle immagini.


p.s.
comunque, credo di aver individuato come procedere

ShadowMan
27-01-2010, 16:46
Basta semplicemente mettere in evidenza x ln 2:
così ti esce [x ln2](1 -x^(1/3)/(xln2)]
Ovviamente la radice quadrata è un o piccolo di x per x-->+ infinito, da ciò l'espressione.

Grazie per la risposta ma ci sono delle cose che non mi sono chiare:
di quale radice quadrata parliamo? :stordita:
x tende a -oo [ meno infinito] non +oo

Pensandoci, io ho un radice cubica / x * costante.
Con x che tende a -oo entrambe tendono a -oo ma facendo un confronto tra infiniti vince X, quindi ottengo 0.

Jarni
27-01-2010, 17:19
Grazie per la risposta ma ci sono delle cose che non mi sono chiare:
di quale radice quadrata parliamo? :stordita:
x tende a -oo [ meno infinito] non +oo

Pensandoci, io ho un radice cubica / x * costante.
Con x che tende a -oo entrambe tendono a -oo ma facendo un confronto tra infiniti vince X, quindi ottengo 0.

x^(1/3)
--------
x

Questa espressione tende a 0 sia se x tende a +inf sia se tende a -inf semplicemente perché è uguale a

1
-------
x^(2/3)

ShadowMan
27-01-2010, 17:34
Ok, ora è più chiaro grazie. :)

kierlo
27-01-2010, 19:44
Eh, per la fretta mi ero confuso..^^ sorry

ndakota
28-01-2010, 11:37
Ciao a tutti, qualcuno mi toglie qualche dubbio sui simboli di landau? Se per esempio ho
An = 3^n
Bn = sqrt(n!)

che simbolo usereste per metterli in relazione? Io ho pensato che An = O(Bn) oppure Bn = o(An)
però non sono affatto sicuro. Voglio dire, io penso che per quanto il fattoriale possa essere grande poi c'è una radice che essendo infinito potenza è minore di un infinito esponenziale. Sbaglio? Sì :D

gugoXX
28-01-2010, 11:47
Dunque, sappiamo che il logaritmo "perde" sempre contro una qualsiasi potenza

http://operaez.net/mimetex/%5Clim_%7Bx=%3E%5Cinfty%7D%5Cfrac%7B%5Cln%7Bx%7D%7D%7Bx%7D=0


Ma anche quando la potenza e' molto piccola, seppur >0 ?

http://operaez.net/mimetex/%5Clim_%7Bx=%3E%5Cinfty%7D%5Cfrac%7B%5Cln%7Bx%7D%7D%7Bx%5Eb%7D
(Con b>0, ma molto piccolo, es: b=0.00001)

Ma anche una qualsiasi potenza del logaritmo perde sempre?
http://operaez.net/mimetex/%5Clim_%7Bx=%3E%5Cinfty%7D%5Cfrac%7B%5Cln%5Ea%7Bx%7D%7D%7Bx%5Eb%7D
(con b>0, ma molto piccolo, e a>0, ma molto grande)

E questa? Anche se si passa per l'Hopital a calcolarla e' brutto...
http://operaez.net/mimetex/%5Clim_%7Bx=%3E%5Cinfty%7D%5Cfrac%7B%5Cln%5Ex%7Bx%7D%7D%7Bx%7D

ndakota
28-01-2010, 12:57
gugo, che piacere vederti qui :flower: ti confesso che non ho capito se il tuo post è una risposta al mio o meno :D

gugoXX
28-01-2010, 13:13
gugo, che piacere vederti qui :flower: ti confesso che non ho capito se il tuo post è una risposta al mio o meno :D

Ahah. Nono. E' che ho trovato un vecchio libro di analisi e sto provando a fare esercizi a tempo perso, per non perdere allenamento.
Tanto piu' che dovro' a breve seguire un corso in cui la matematica e' richiesta a livello analisi I almeno, quindi sto cercando di ripassare.

ndakota
28-01-2010, 13:25
Ahah. Nono. E' che ho trovato un vecchio libro di analisi e sto provando a fare esercizi a tempo perso, per non perdere allenamento.
Tanto piu' che dovro' a breve seguire un corso in cui la matematica e' richiesta a livello analisi I almeno, quindi sto cercando di ripassare.

Ah ok, allora aspetto mi chiarisca qualcun altro, anche se ho già visto con wolfram che è il contrario di quello che dicevo io :muro:

Per il tuo invece http://www.wolframalpha.com/input/?i=limit%28%28lnn%29^n%2Fn%29%2C+n-%3E%2Binf

gugoXX
28-01-2010, 13:35
Ciao. Direi che vince la radice del fattoriale.
Secondo Stirling la radice di fattoriale va come e^(N log N), con la radice siamo a E^(N/2 Log N).
Mentre l'altra e' (E ^ 3 Log N)

Jarni
28-01-2010, 13:42
Ciao a tutti, qualcuno mi toglie qualche dubbio sui simboli di landau? Se per esempio ho
An = 3^n
Bn = sqrt(n!)

che simbolo usereste per metterli in relazione? Io ho pensato che An = O(Bn) oppure Bn = o(An)
però non sono affatto sicuro. Voglio dire, io penso che per quanto il fattoriale possa essere grande poi c'è una radice che essendo infinito potenza è minore di un infinito esponenziale. Sbaglio? Sì :D

Ragionando su due piedi, fai il rapporto An/Bn e elevi al quadrato(lo puoi fare senza perdere di generalità):

9^n
-------
n!

capirai che da un certo numero in poi avrai

9*9*9*9*9*9*9*9*9*9...
--------------------------------------------------------------------------------
(numero enorme)*(numero enorme+1)*(numero enorme+2)*(numero enorme+3)...

A occhio vince il denominatore.:D

jacky guru
28-01-2010, 14:51
EDIT, solved :)

ndakota
28-01-2010, 15:06
Come procedereste per determinare il numero esatto di soluzioni di qualcosa tipo

2x + 1 = 4 acrtan x

? Vedendo i grafici si incontrano due volte, quindi dovrebbero essere due soluzioni, ma come ci si arriva? Perchè facendo i grafici separatamente di 2x + 1 e di 4 arctan x è davvero ardua.

gugoXX
28-01-2010, 15:14
Ah ok, allora aspetto mi chiarisca qualcun altro, anche se ho già visto con wolfram che è il contrario di quello che dicevo io :muro:

Per il tuo invece http://www.wolframalpha.com/input/?i=limit%28%28lnn%29^n%2Fn%29%2C+n-%3E%2Binf

Grazie.

Ziosilvio
28-01-2010, 17:12
Ciao a tutti, qualcuno mi toglie qualche dubbio sui simboli di landau? Se per esempio ho
An = 3^n
Bn = sqrt(n!)

che simbolo usereste per metterli in relazione? Io ho pensato che An = O(Bn) oppure Bn = o(An)
però non sono affatto sicuro. Voglio dire, io penso che per quanto il fattoriale possa essere grande poi c'è una radice che essendo infinito potenza è minore di un infinito esponenziale. Sbaglio? Sì :D
Beh, 3^n = (sqrt(9))^n = sqrt(9^n), quindi:

http://operaez.net/mimetex/%5Cfrac{3^n}{%5Csqrt{n!}}=%5Csqrt{%5Cfrac{9^n}{n!}

che converge a...

gugoXX
28-01-2010, 17:39
Come procedereste per determinare il numero esatto di soluzioni di qualcosa tipo

2x + 1 = 4 acrtan x

? Vedendo i grafici si incontrano due volte, quindi dovrebbero essere due soluzioni, ma come ci si arriva? Perchè facendo i grafici separatamente di 2x + 1 e di 4 arctan x è davvero ardua.

Ti va bene che devi calcolare solo il numero di soluzioni, non dove.

Prendi Atan(x), calcolane la derivata e guarda dove vale 2.
Dovresti ottenere 2 punti.
Calcola le 2 rette aventi inclinazione 2 passanti per tali punti, otterrai 2 punti in cui queste 2 rette incroceranno l'asse Y.

Se il punto in cui la tua retta incrocia l'asse Y e' compreso tra questi 2 punti, avrai necessariamente 3 soluzioni, essendo la tua retta inclinata solo 2, mentre l'ATN inclinata di 4 nell'origine.

Se e' oltre, avrai una sola soluzione (negativa, con incrocio nel 3 quadrante)
Se e' piu' basso, avrai una sola soluzione (positiva, con incrocio nel 1 quadrante)
Se e' esattamente uno dei punti di cui prima, avrai invece due soluzioni coincidenti, proprio nel punto in cui avevi trovato la tangente (della quale saprai anche perfettamente la posizione quindi), e un'altra soluzione.

misterx
28-01-2010, 18:21
edit

ndakota
28-01-2010, 19:33
Beh, 3^n = (sqrt(9))^n = sqrt(9^n), quindi:

http://operaez.net/mimetex/%5Cfrac{3^n}{%5Csqrt{n!}}=%5Csqrt{%5Cfrac{9^n}{n!}

che converge a...

0? :stordita:

da quel che so io
log n = o(x^a) , a > 1
x^a = o(q^x) , q > 1
q^x = o(n!)
n! = o(n^n)

giusto?

Ziosilvio
28-01-2010, 21:01
0? :stordita:

da quel che so io
log n = o(x^a) , a > 1
x^a = o(q^x) , q > 1
q^x = o(n!)
n! = o(n^n)

giusto?
Giusto se scrivi sempre n al posto di x e se f=o(g) vuol dire f(n)/g(n)-->0 per n-->oo.

ndakota
28-01-2010, 21:30
Giusto se scrivi sempre n al posto di x e se f=o(g) vuol dire f(n)/g(n)-->0 per n-->oo.

Sì certo :p

mi è meno intuitivo capire O(o grande), omega grande e theta grande. Su asintotico e o piccolo ci sono.

Dico proprio dalla definizione. Che poi vabbè, sbaglio o sono tre casi? Sono legati tra loro diciamo i simboli "grande".

ShadowMan
29-01-2010, 11:27
I conti non tornano :asd:

Io so che il polinomio di taylor di grado 2 di log(x+1) è uguale a T= x - 1/2 x^2

Ora, io devo calcolarmi T2 di log(x^2+x+1).
Poiché x^2+x -> 0 per x->0 posso sostituirlo nella formula sopracitata ed ottengo:
x^2 + x -1/2x^4 - 1/2x^2 = x + 1/2 x^2 - 1/4 x^4

Guardo la soluzione e vedo x + 1/2 x^2. :cry:
Cosa ho sbagliato ? :stordita:

jacky guru
29-01-2010, 14:34
I conti non tornano :asd:

Io so che il polinomio di taylor di grado 2 di log(x+1) è uguale a T= x - 1/2 x^2

Ora, io devo calcolarmi T2 di log(x^2+x+1).
Poiché x^2+x -> 0 per x->0 posso sostituirlo nella formula sopracitata ed ottengo:
x^2 + x -1/2x^4 - 1/2x^2 = x + 1/2 x^2 - 1/4 x^4

Guardo la soluzione e vedo x + 1/2 x^2. :cry:
Cosa ho sbagliato ? :stordita:

A mio avviso nulla :stordita: Hai semplicemente omesso di "inglobare" 1/4x^4 nell'o-piccolo di x^2 :)

ShadowMan
29-01-2010, 14:41
A mio avviso nulla :stordita: Hai semplicemente omesso di "inglobare" 1/4x^4 nell'o-piccolo di x^2 :)

Era quello che avevo pensato, ma mi è sorto il dubbio poiché nella soluzione non c'è traccia di o(x^2). :mc:

Perché sul libro quando parla degli sviluppi taylor non c'è traccia di o(x^n), li usa solo quando approssima una funzione con i polinomi di taylor. :stordita:

jacky guru
29-01-2010, 15:04
Era quello che avevo pensato, ma mi è sorto il dubbio poiché nella soluzione non c'è traccia di o(x^2). :mc:

Perché sul libro quando parla degli sviluppi taylor non c'è traccia di o(x^n), li usa solo quando approssima una funzione con i polinomi di taylor. :stordita:
LOL sicuro? Che io sappia non esiste nessuna funzione polinomiale che approssima ESATTAMENTE al 1000% una funzione data: per questo nello sviluppo di DEVE essere un o-piccolo, che indica proprio lo scarto di ordinate, ovvero la distanza tra le due curve, nell'intorno del punto rispetto al quale calcoli lo sviluppo... chiedo venia in anticipo per il mio matematichese maccheronico :p

ShadowMan
29-01-2010, 15:34
bhe...sul libro c'è scritto così :asd:
Penso lo dia per scontato. :stordita:

matrix866
29-01-2010, 16:21
Come si risolve questo integrale????? Vedi foto allegata! Grazie!

P.S. Nella foto manca il dx.

jacky guru
29-01-2010, 17:16
bhe...sul libro c'è scritto così :asd:
Penso lo dia per scontato. :stordita:
Mmhh ma è un libro delle superiori o "universitario"?
Come si risolve questo integrale????? Vedi foto allegata! Grazie!

P.S. Nella foto manca il dx.
Ciao, dovrebbe bastare sostituire e^x con t, in questo caso dovrebbe venirti un integrale di funzione razionale fratta facile facile :)

ShadowMan
29-01-2010, 17:55
Universitario, comunque sono certo che da per scontato gli o piccoli anche perché è l'unica spiegazione che riesco a darmi. :sofico:

SerMagnus
29-01-2010, 18:48
Raga so che sto per fare una gigantesca figura di m***a :D ma mi sono incartato con questa disequazione... sarà che sto fuso:

4x^2+8x>=0
io mi trovo x<=0 e x>=-2
ma sono andato a verificare e invece il risultato corretto è x>=0 e x<=-2

dove cribbio sbaglio :muro: :muro: :muro:

misterx
29-01-2010, 19:19
4x^2+8x>=0

raccolgo una x in quanto la disequazione essendo spuria (http://www.ripmat.it/mate/a/af/afcb.html)e mi consente di farlo
x(4x + 8)=0
x1=0 per la legge di annullamento del prodotto
x2=-2

+++++++++++ (-2) --------------- (0) ++++++++++++++

x <= -2 v x >= 0

misterx
29-01-2010, 19:23
ciao,
come si riconduce questo limite ad un limite notevole ?


lim [(n^2 - n) / (n^2 + 3)]^n
n-> +oo


dovrebbe essere una forma di indecisione del tipo 1^inf

Tralasciando De l'Hopital, come ci si arriva come ho già detto modellando il limite dato rendendolo simile ad un limite notevole ?

grazie

ndakota
29-01-2010, 19:31
ciao,
come si riconduce questo limite ad un limite notevole ?

[code]
lim [(n^2 - n) / (n^2 + 3)]^n
n-> +oo
[code]

dovrebbe essere una forma di indecisione del tipo 1^inf

Tralasciando Del l'Hopital, come ci si arriva come ho già detto modellando il limite dato rendendolo simile ad un limite notevole ?

grazie

+3-3 sopra?

Così dovresti ritrovarti con

lim [1 + (-3n)/(n^2+3)]^n
n-> +oo

e poi continui.. insomma dovrebbe essere quello

lim (1 + 1/n)^n = e
n->+oo

misterx
29-01-2010, 19:42
+3-3 sopra?

Così dovresti ritrovarti con

lim [1 + (-3n)/(n^2+3)]^n
n-> +oo

e poi continui.. insomma dovrebbe essere quello

lim (1 + 1/n)^n = e
n->+oo


scusa ma non ho capito l'idea che hai avuto

ciao

ndakota
29-01-2010, 19:46
Sommando e sottraendo 3 a numeratore ti ritrovi con

[(x^2 + 3)/(x^2 + 3) + (-3x)/(x^2 + 3)]^n

--------------------
----------1

che è molto simile a quel limite notevole, no? Non capisci come andare avanti o era questo passaggio?

misterx
29-01-2010, 20:03
Sommando e sottraendo 3 a numeratore ti ritrovi con

[(x^2 + 3)/(x^2 + 3) + (-3x)/(x^2 + 3)]^n

--------------------
----------1

che è molto simile a quel limite notevole, no? Non capisci come andare avanti o era questo passaggio?


non avevo capito la tecnica risolutiva, a quanto ho capito devi avere occhio. Avendo ora (1-epsilon)^n in luogo di (1+epsilon)^n = e, come ci si deve comportare ?
C'è il segno meno che non mi quadra.

ciao

ndakota
29-01-2010, 20:09
lasci + e il meno lo lasci solo nella frazione, il passaggio dopo dovrebbe essere

lim [1 + (-3n)/(n^2+3)]^(n^2+3)/(-3n)*n
n-> +oo

ora possiamo applicare il limite notevole e quindi dovrebbe rimanere

lim e^n = +oo
n->+oo

ndakota
29-01-2010, 20:12
cancella tutto.. a quanto pare è sbagliato :D
http://www.wolframalpha.com/input/?i=limit%28%28%28n^2-n%29%2F%28n^2%2B3%29%29^n%29%2C+n-%3E%2Binf

misterx
29-01-2010, 20:14
cancella tutto.. a quanto pare è sbagliato :D
http://www.wolframalpha.com/input/?i=limit%28%28%28n^2-n%29%2F%28n^2%2B3%29%29^n%29%2C+n-%3E%2Binf

ma il risultato lo conosco, però mi interessa come ci si arriva

ciao

ndakota
29-01-2010, 20:16
ma il risultato lo conosco, però mi interessa come ci si arriva

ciao

Eh ma a quanto pare il mio è sbagliato :sofico:

Deve sfuggirmi qualcosa perchè il mio professore quelli di questo tipo li ha sempre fatti così :stordita:

misterx
29-01-2010, 20:28
Eh ma a quanto pare il mio è sbagliato :sofico:

Deve sfuggirmi qualcosa perchè il mio professore quelli di questo tipo li ha sempre fatti così :stordita:


con quel meno (1 - 1/x)^x -> 1/e, basta capire come salta fuori

ciao

ndakota
29-01-2010, 20:57
con quel meno (1 - 1/x)^x -> 1/e, basta capire come salta fuori

ciao

no edit

kierlo
30-01-2010, 09:10
se hai (1-E)^1/E (con E intendo Epsilon n) basta pensarlo come

(1+(-En)^ (-1/-E) e semplificando ti esce e^-1 ossia 1/e

misterx
30-01-2010, 09:39
se hai (1-E)^1/E (con E intendo Epsilon n) basta pensarlo come

(1+(-En)^ (-1/-E) e semplificando ti esce e^-1 ossia 1/e


ciao,
dopo i vari passaggi io ho (1-E)^x

ShadowMan
30-01-2010, 09:50
lim (cos x) ^ 1/|x| con x->0
so che è uguale a e^[lim 1/|x| (ln cos x)] con x->0

cos x con x->0 posso scriverlo come 1 - 1/2 x^2 + o(x^3)

log (cos x) = log ( 1 + (cos x -1)) = (cos x -1) - 1/2 (cos x - 1)^2 + o((cos x -1)^2)

-> log ( 1 + (cos x -1)) = -1/2 x^2 + o(x^3) - 1/2( -1/2x^2 + o(x^3))^2 + o(x^3) = - (x^2)/2 + o(x^3)

Quindi 1/|x| (- (x^2)/2) = 0 per x->0.
e^0 = 1 quindi il limite vale 1? :stordita:

Ponga92
30-01-2010, 10:52
avrei bisogno di un aiuto:
a scuola abbiamo fatto l'eq dell'ellisse partendo da una circonferenza:
circ: x^2+y^2-10x+4y+28=0
per passare dalla circonferenza all'ellisse abbiamo posto
x'=2x y'=4y (non sono le derivate)
sostituendole all'eq della circonferenza viene
4x^2+y^2-80x+16y+448=0

Però utiliazzando la formula dell'ellisse traslata viene
http://img10.imageshack.us/img10/7698/codecogseqn4.png

Vorrei sapere come mai le due equazioni non vengono uguali.

kwb
30-01-2010, 11:06
Ho da risolvere l'integrale di:
x^2/(x^3-x^2+x+1)

Come devo procedere per l'identità polinomiale?

misterx
31-01-2010, 09:37
ciao,
qualcuno riesce ad intuire come si è giutni a far sparire la radice ?
La prima riga e cioè il primo passaggio è chiaro ma il secondo e cioè la seconda riga di conti partendo dal basso ?
grazie
http://i49.tinypic.com/2mrxfk0.gif

Lampo89
31-01-2010, 11:47
nella seconda riga si è sviluppato entrambe le radice con taylor arrestato al primo ordine : per n tendente a +inf vale (1+1/n)^alfa = 1 + alfa*1/n +o(1/n) . poichè l'infinitesimo dominante è 1/n (infatti 1/n^3 è un infinitesimo di ordine superiore) i termini 1/n^3 e o(1/n^3) vengono " mangiati" dall'o(1/n) e si possono trascurare.

kierlo
31-01-2010, 12:08
In caso tu non abbia fatto taylor, non è altro che un limite notevole:

(1+f(x))^a=1+a*(f(x)).

Esclusivamente per f(x)->0

misterx
31-01-2010, 13:32
nella seconda riga si è sviluppato entrambe le radice con taylor arrestato al primo ordine : per n tendente a +inf vale (1+1/n)^alfa = 1 + alfa*1/n +o(1/n) . poichè l'infinitesimo dominante è 1/n (infatti 1/n^3 è un infinitesimo di ordine superiore) i termini 1/n^3 e o(1/n^3) vengono " mangiati" dall'o(1/n) e si possono trascurare.


"o piccolo" a parte, credo di avere capito: grazie

misterx
31-01-2010, 13:33
In caso tu non abbia fatto taylor, non è altro che un limite notevole:

(1+f(x))^a=1+a*(f(x)).

Esclusivamente per f(x)->0


lo sto ripassando, grazie

misterx
31-01-2010, 17:29
scusate ma per applicare Taylor devo calcolare la derivata prima delle radici giusto ?
Nell'immagine mancano parecchi passaggi :(
http://i49.tinypic.com/2mrxfk0.gif

ShadowMan
31-01-2010, 17:57
Di che deviata parli? :mbe: :confused: :stordita:

radice cubica di (1 - 1/n^3) = (1 - 1/n^3) ^ 1/3

Quindi applichi il procedimento illustrato da Lampo89.

misterx
31-01-2010, 19:06
ciao,
se applichi Taylor devi derivare

f(x0) + f'(x0)(x - x0)+ f"(x0)(x - x0)^2/2! + f'''(x0)(x - x0)^3/3! + ... + f(n)(xn)(x - x0)^n/n!

ShadowMan
31-01-2010, 19:23
uhmm, non devi applicare quello.

1/n ->0 per n->inf, usi macluarin [cioè sviluppo di taylor in 0] di cui ci sono gli sviluppi noti.

cuberoot(1 - 1/n^3) = (1 - 1/n^3)^1/3 = (1 + y)^1/3

lo viluppo di maclaurin (1 + y) ^a = 1 + ay + o(y) [lì si fermato al primo ordine]

(1 + y)^1/3 = 1 - 1/3 (1/n^3) + o(1/n^3)

Stesso procedimento per l'altra radice solo che lì y=1/n